منتدي نجم السودان
د.نجم الدين علي ياسين
يتمني لكم اقامه طيبه داخل صفحات المنتدي
ارجو التسجيل معنا حتي تتمكن من الاستمتاع بكل المزايا
منتدي نجم السودان

MCQS Part 1 MRCOG More than 300 stem 13458470.b7d6c5

Join the forum, it's quick and easy

منتدي نجم السودان
د.نجم الدين علي ياسين
يتمني لكم اقامه طيبه داخل صفحات المنتدي
ارجو التسجيل معنا حتي تتمكن من الاستمتاع بكل المزايا
منتدي نجم السودان

MCQS Part 1 MRCOG More than 300 stem 13458470.b7d6c5
منتدي نجم السودان
Would you like to react to this message? Create an account in a few clicks or log in to continue.

MCQS Part 1 MRCOG More than 300 stem

+9
رهام
ahmed bassett
drmamou
mujtaba
aml
drhmada
wafyazaz
dr-dina
mandible
13 posters

Go down

MCQS Part 1 MRCOG More than 300 stem Empty MCQS Part 1 MRCOG More than 300 stem

Post by mandible Sun May 23, 2010 2:00 am

[You must be registered and logged in to see this link.]

[PART 1 MRCOG FINAL ASSESSMENT
1.Which of the following will be seen in a villous of the placenta :
a. syncytiotrophoblast
b. cytotrophoblast
c. yolk
d. decidual cells
e. blood vessels

2. as pregnancy progresses, the amniotic fluid :
a. become more hypotonic
b. concentration of Na will falls
c. urea concentration increases
d. LFP concentration increase
e. PH become more acidic

3. These methods of placenta transfer are appropriate for the following substances :
a. oxygen _______________ active
b. fats __________________pinocytosis
c. iron __________________active
d. co2 ___________________ passive
e. antibodies _______________ pinocytosis

pharmacology

4. morphine :
a. the major pathway for it is detoxification is by oxidation to gentisic acid
b. increase biliary tract pressure
c. is eliminated mainly in the conjugated state via glomerular filtration
d. is secreted in the bile
e. produce excitement in male and female equally it does.

5. codeine
a. is not effective in large doses for severe pain
b. large doses cause excitement
c. large dose causes narcosis
d. is strong anittusive agent
e. aspirin and codeine are a useful combination against headache

6. the hazards of induction of anesthesia by injection of thiopentone are :
a. severe hypotension
b. cessation of respiration
c. necrosis following extra vascular injection
d. bronchospasm
e. stimulation of bronchial and salivary secretions

:
7. pethidine:
a. has analgesic properties .
b.is effective for biliary colic
c.will often relieve bronchial spasm
d.has quinidine like effect on myocardium
e. may cause hypotension

8. halothane :
a. is potent anesthetic and poor analgesic
b. cause uterine relaxation
c. cause liver failure in biliary tract disease
d. does not increase the catecholamine secretion
e. cause increased blood loss in therapeutic abortion


9. the gynecoid pelvis :
a. is the second commonest type of female pelvis
b. has wider anteroposterior than transverse diameter at the pelvic prim
c. has less prominent ischial spines than android pelvis
d. has ashallow ,wide sacrosciatic notch
e. has a parallel pelvic side wall .

10. the capsule of the hip joint :
a. contain fibrous bands which convey nutrient arteries supplying neck of femur
b. is strongest anteriorly and superiorly
c.is strongest posterior-inferior
d. cover the neck of the femur completely

11. the sacroiliac joint :
a. is asynovial joint
b. its main bond is the dorsal sacroiliac ligament
c . the interossious ligament is the strongest ligament of the joint
d. has anterioposterior rotary ligament
e. movements here are impaired by pregnancy

12. which of the following statements are correct in relation to the ischial spines :
a. they lie between the greater and lesser sciatic notches
b. they mark the begging of the forward curve of the birth canal
c. they are of particular prominence in platypelloid pelvis
d. when the head at its level , they consider that head is not yet engaged .
e.the internal pudendal nerve lies in close relationship to the spines

13. the following structures pass below the piriformis muscle through the greater sciatic notch :
a. sciatic nerve
b. the lateral femoral cutaneous nerve
c. the pudendal nerve
d. the nerve to the obturator externus
e. the inferior gluteal vessels

14. an ideal pelvis has :
a. a straight sacrum
b. an angle of inclination of 135
c. a subpubic angle of about 70
d. palpable sacral promontory .
e. parallel pelvic side walls


[You must be registered and logged in to see this link.]

Embryology

15. chromatin negative conditions include :
a. xx
b. xo
c. xxy
d. xxx
e. xy

16.differentiation of sexual organs :
a. is primarily directed by genetic factors
b. is influenced by supra renal cortex
c. is influenced by N((0)) of primordial germ cells
d. is influenced by over ripeness of ovum at fertilization
e. is not affected by chromosomal constitution

17. male and female analogues :
a. testis correspond to ovary
b. prostatic utricle corresponds to vagina
c. ventral penis corresponds to labia majora
d. distal male urethra corresponds to female vestibule
e. penis corresponds to crus clitoridis

18. the barr body :
a. arise from the Y chromosome
b. is found in turner syndrome
c. is found in testicular feminization S
d. occurs in 10 % of normal male
e. is found near the cell membrane on microscopic examination.

19. drugs which affect the fetus in late pregnancy :
a. oxytetracycline
b. long acting sulfa
c. streptomycin
d. ampicillin
e. erythromycin

20. cytotoxic drugs :
a. antimetabolites include methotrexate 5FU, 6 mercaptopurine
b. methotrexate block folic reductase
c. alkaylating agents include thiotepa and synthetic progestins
d. cycophosamide causes hemorrhagic cystitis
e. vincristine causes peripheral neuropathy

21. the drug of choice for typhoid fever :
a. sulphadiazine
b. penicillin
c. chloramphenicol
d. tetracycline
e. neomycin

22. the following karyotypes are appropriate :
a. turners syndrome 46 XO
b. testicular feminization 47 XY
c. klinfelters syndrome 47 XXY
d. super female 46 XXX
e.down syndrome 47 trisomy 21

23. ambiguous external genitalia at birth :
a. are commonly due to congenital adrenal hypoplasia
b. are associated with drug ingestion during pregnancy
c. occurs in testicular feminization syndrome
d. occurs in true hermaphroditism
e. are commonly associated with abnormal karyotype

24. the following effective against penicillinase producing staph :
a.cloxacillin
b. carboncillin
c. ampicillin
d. phenoxy methyl penicillin
e.flagyl

25. the following items are useful for prevention of side effect of chemotherapy :
a. full body irradiation
b.folinic acid
c. scalp cooling
d. corticosteroids
e. leucocytes transfusion

[You must be registered and logged in to see this link.]



26. the clotting mechanism :
a. the clotting factors may be assayed chemically
b. thrombin converts fibrinogen to fibrin monomer by splitting fibrinopeptides A + B
c. thrombin alters antihaemophilic factor VIII
d. thrombin is important in platelets haemostatic behavior
e. platelets accelerate the intirinsic clotting pathway by providing Ca
27.estrogen metabolism by the liver :
a. all the estrogens are inactivated by the liver
b. estrogen is 16 alpha- hydroxylated to 16 alpha hydroxytesterone
c. excretion in the bile is either in the conjugated or free form
d. in the liver cellular disease , impaired hydroxylation occur
e. urinary estrogen determination is a useful liver function test

[You must be registered and logged in to see this link.]

28. cholesterol :
a. is a constituent of most cell membranes
b. is a precursor of steroid only
c. synthesis take place mainly from acetate
d. synthesis occurs in microsomes and cell supply
e. bile salts are important un the regulation of its metabolism

29. amniotic fluid contains the following protein :
a. alpha 1 globulin
b. alpha 2 globulin
c. B globulin
d. Y globulin
e. all the above

30. normal haemopoiesis requires the following elements of vitamins :
a. iron
b. folic acid
c. cyanocobaamine
d. copper
e. manganese

31. is it true that :
a. insulin promote the uptakes of glucose by muscle cells
b. glucose 6- phosphatase occurs in muscle but not in liver
c. adrenalin promotes the breakdown of hepatic glycogen
d. lactate is the end product of the aerobic metabolism of glucose
e. insulin is secreted by the alpha cells of the islets of langerhans

32. which of the following are correct :
a. DNA is found in the nucleus and cytoplasm
b. RNA is found in the nucleus only
c. RNA is found in the cytoplasm only
d. RNA is found in the cytoplasm and nucleus
e. DNA is the chief constituent of ribosomes

33. which of the following are active process :
a. Ca ++ transport in the sarcoplasmic reticulum
b. Hcl secretion in the gastric epithelium
c. amino acid transport in the gut
d.glucose transport in the erythrocyte
e. C1/HCO exchange in erythrocytes


[You must be registered and logged in to see this link.]

34. statistics :
a. the median is a value such that 50 % of the observation fall above it and 50 % below it
b. the mode describes the most frequent value
c. the standard deviation measures scatter of distribution
d. the variance measures the greatest spread
e. sequential trials allow a trial to be discontinued at the earliest moment at which a judgment can be objectively formed

35. the standard deviation of a group of observation :
a. is the square of variance of this group
b. is a measure of the scatter observation around the mean
c. has a normal Gaussian distribution
d. may be used as the basis of the calculation of X 2
e. about 91 % of observation lie within 2 standard deviation of the mean




36.radiation
a. beta rays may be +ve or –ve electrone
b. gamma rays are similar to x-ray
c.a roentgen is equal to 48 ergs/gm
d. a rad measures emitted radiation
e. a rad is equal to 100 ergs for energy/gm

37. Radiation: the following isotopes have the half lives:
a. cesium 137 - 15 years
b. cobalt 60 - 5.2 years
c. radium 226 - 158 years
d. radium 222 - 1620 years
e. strontium 90 – 25 years

38. Respiratory changes in pregnancy:
a. with increasing pressure , the ribs flare out
b. the normal 168 subcostal angle increase to +_ 103 at turn
c. the diaphragm rise by +_ 4 cm
d.the transverse diameter of the chest increase by +_ 2 cm
e. respiratory is mainly diaphragmatic in pregnancy.
[You must be registered and logged in to see this link.]

39. the following respiratory functions increase in pregnancy:
a. vital capacity
b. tidal volume
c. residual volume
d. inspiratory capacity
E. minute ventilation

40. Cardiac action in pregnancy:
a. the mean COP is +_ 61/min
b. COP rises by +_1.5 L/M during 1st 10 weeks
c. COP rises by 1/3 during pregnancy
d. pulse rate rise by 1/5 during pregnancy
e. stroke volume falls during pregnancy

41. Effects of changed respiratory functions in pregnancy:
a. blood pH falls in pregnancy
b. plasma bicarbonate fall
c. plasma Na level fall
d. plasma osmolality fall
e. dyspnea is common even in absence of pathology

42. Erythrocytes & haemoglobin in pregnancy:
a. RBC fall from the average non – pregnant level of +_4.5 mill/ml to a minimum of +_3.5 mill/ml at about 30 weeks
b. the RBC count rises after 30 weeks
c. the hematocrit rises progressively to 38 weeks
d. erythrocytes tend to be more spherical in shape
e. Hb falls from 137 – 140 g/L to 110 – 120 g/L

43. Total body water:
a. may be as low as 30 % of body weight in newborn
b. is greater in male than female
c. is one half to 2/3 of body weight
d.is measured by dentrium oxide
e. is less in thin people

44. Lymphatics:
a. are characterized by their fast flow
b. are lined by endothelial cells
c. are permeable to protein
d. are scarce in cellular tissue
e. of intestine , transport and absorbed fat

45. Na reabsorption in proximal convoluted tubules:
a. is under hormonal control
b. is a passive process
c. is related to K secretion
d. is accompanied by passive absorption of CL
e. increased in pregnancy .

46. Which of the following dietary component are most likely to be inadequate in human pregnancy?
a. ascorbic acid
b. vitamin E
c. Ca++
b. Nacl
e. Iron

47. Oxygen dissociation curve shifts to left with:
a. raised temp
b. raised PCO2
c. reduced PH
d. anemia
e. stored blood

48. Hyperventilation:
a. in necessarily followed by period of apnea
b. decreased H2CO3 in the plasma
c. leads to tetany
d. increased Po2 of arterial blood
e. correct pH in metabolic acidosis

49. Veins:
a. of lower limbs have no valves
b. of neck are never collapse in erect position
c. are thin walled
d. Are innervated
e. of upper limb have no valves

50. Insulin cause:
a. increase utilization of glucose by the cells
b. increase glycogen store by the liver
c. decrease the amino acids and level in blood
d. decrease protein synthesis
e. decrease fatty acids levels in the blood

51. in respiratory acidosis:
a. the blood plasma reaction is acid
b. the kidney excrete an alkaline urine
c. the plasma bicarbonate is decreased
d. the primary defect is hypoventilation
e. cause can be ventilation/perfusion imbalance

52. Toxoplasmosis:
a. the dog is the definite host
b. the parasite exist in 2 forms cyst & oocyst
c. invasion is by the cyst
d. the oocyst is also important in human infection
e. cause cervical lymphadenopathy

53. The following diseases have an incubation period less than 10 days:
a. rabies
b. filariasis
c. dengue
d. yellow fever
e. hepatic amoebiasis

54. Anaerobic bacteria:
a. include gm +ve as well as gm –ve organism
b. produce hyaluronidase
c. bacteroid fragilis is most commonly seen
d. corticosteroid therapy lessen the chance of infection
e. the anaerobic gm –ve bacilli & anaerobic coccci are the major agent in aerobic infection
55. Hepatitis B antigen:
a. consist of Hbs Ag fraction
b. consist of Hbc Ag fraction
c. is only found in the serum sufferers (2-6) weeks after evidence of transaminase activity
d. occurs in 1/10000 of Caucasian
e. may spread in faeces

56. Structure of the simpler virus:
a. in the simpler viruses , the mature virus particle is the virion
b. the protein coat is the capsid
c. the centre is a single molecule of nuclei A
d. the nucleocapsid is the nucleic acid core plus the protein coat
e. capsids are composed of repeating units of one or a small number of polypeptides .

57. the agents of first choice for the organism mentioned are :
a. N. gonorrhea : cephalothin
b. T. palladium : tetracycline
c. C. wechiu : penicillin G
d. E. coli : gentamycin
e. S. aureus : methicillin, oxacillin or norfloxacin

58. consider cell necrosis , a nucleus which shrivels and become more dense is described as:
a. karyorhexis
b. karyolysis
b. pyknosis
d. vacular degeneration
e. condensation

59. Mismatched blood transfusion may lead to:
a. haemoglobinemia
b. haemoglobiuria
c. bilirubinuria
d. polycythaemia
e. renal failure

60. The characteristic reaction to be found in a tubercle usually includes:
a. pus cells
b. siderocyte
c. foreign body giant cells
d. epithelioid cells
e. lymphocytes

61. Multinucleated giant cells are seen in man in association with:
a. cat-scratch disease
b. measles
c. rheumatic fever
d. syphilis
e. TB

65. Necrosis is an important feature of the lesion in:
a. acute anterior poliomyelitis
b. labour pneumonia
c. rheumatic fever
d. tertiary syphilis
e. viral hepatitis

66. the following are true of a normal distribution :
a. 95 % of observations lie within mean +- 1 standard deviation (SD)
b. 2.5 % of observations lie below 2 standard deviation of the mean.
c. student s t-test could be used to compare the date with those of another population which is normally distributed
d. anon-parametric test may may be used to compare the data with those of another population .
e. 99 % of observational lie within 2.6 standard errors of the mean

67. The following statements are correct:
a. the standard error of the mean (SEM) provides an index of the spread of observations around the mean
b. the standard error of the mean is calculated as the square root of variance
c. the standard deviation is generally smaller than the standard error of the mean
d. the standard deviation is an index of the reliability of the mean
e. one advantage of the standard deviation is that it could be manipulated mathematically

68. Hyperpigmenation can occur in pregnancy in the following sites:
a. nipples and areolae
b. umbilicus
c. axillae
d. perineum
e. face
69. Alkalinization of the urine will enhance the excretion of:
a. phencyclidine
b. amiphetamine
c. phenobarbitone
d. aspirin
e. imipramine

70. Which of the following drugs may be administered orally with tetracycline?
a. magnesium trisilicate
b. aluminum hydroxide
c. salicylates
d. indomethacin
e. ferrous sulphate
71. Cell cycle specific antineuplastic agents act during following diseases:
a. DNA synthesis
b. transcription
c. function of the mitotic spindle
d. predna synthetic phase
e. G0 phase

72. Normal amniotic fluid contains:
a. glucose
b. fructose
c. lactose
d. lactic acid
e. citric acid

73. Concerning the physiology of amniotic fluid:
a. at term , it consist only of fetal urine
b. is removed mainly by fetal swallowing
c. is maximum at term
d. 400 – 500 ml are produced /24hrs
e. 5 ml/kg/hr is produced by a term fetus

74. which of the following separate maternal and fetal blood in human placenta :
a. trophoblast
b. maternal vascular endothelium
c. fetal vascular endothelium
d. maternal connective tissue
e. fetal connective tissue 75. Tinea capitis:
a. is very common in adult
b. can be caused by epidermophyton floccosum
c. can be easy treated topically
d. is common in children than adult
e. can occur in an epidemic

76. Hepatitis A is commonly transmitted by:
a. shared needles of drug users
b. faecal oral transfer
c. airborne respiratory droplets
d. blood transfusion
e. insect vectors
[You must be registered and logged in to see this link.]

77. Human papilloma viruses:
a. are typed by DNA hyperdization
b. are associated with condylomata acuminate
c. rarely cause infection in sexually active men
d. are associated with cervical cancer
e. should be subtyped for identification

78. digestive function in a healthy adult involves :
a. decreased production of saliva following parasympathetic activity
b. voluntary oesophageal contraction
c. release of gastrin from the antral portion of the stomach
d. delayed gastric emptying after a fatty meal
e. increased pepsin secretion with vagal stimulation

79. Monocytes in peripheral blood :
a. are less common than eosinophil granulocytes
b. are more common than basophil granulocytes
c. are phagocytic .
d. may migrate into tissues
e. are decreased in pregnancy

80. when blood vessel is injured :
a. factor XII is activated by collagen
b. factor VII is activated by thromboplastin
c. the intrinsic and extrinsic mechanisms share a common pathway following activation of factor VII
d. tissue factors are responsible for the speed of clot formation
e. the fibrin monomer first formed is soluble in plasma
81. comparing the male and female (gynaecoid ) bony pelvis :
a. the subpubic angle is greater in the female
b. in the female the diameter of the acetabulum is less than the distance between the acetabular rim and the symphysis pubis
c. the interspinous distance is greater in the female
d. the greater sciatic notch is larger in the male
e. the curvature of the sacrum is greater in the female

82. The peritoneum:
a. is lined by stratified squamous epithelium
b. has a visceral layer united firmly with underlying organs
c. has a parietal layer innervated by somatic spinal nerves
d. has venous drainage to the portal vein
e. can function as a dialyzing membrane

83. concerning the rectus sheath :
a. above the costal margin rectus abdominis lies on the costal cartilages
b. below the arcuate line the internal oblique splits to enclose rectus abdominis
c. it contains the musculo-phrenic artery
d. it is innervated by the ilio – inguinal nerve
e. pyramidalis is innervated by the subcostal nerve

84. The following drugs are effective systemically when given by mouth:
a. tetracycline hydrochloride
b. gentamicin sulphate
c. streptomycin
d. chloramphenicol
e. nifedipine

85. The administration of chloroquine phosphate may cause:
a. retinopathy
b. bleaching of the hair
c. changes in the cornea
d. deafness
e. photosensitivity

86. Calcification is associated with:
a. chronic renal failure
b. renal calculi
c. parathyroid adenoma
d. hyaline degeneration in uterine fibroids
e. secondary deposits in bone from prostatic carcinoma

87. collagen
a. is the most abundant protein in the body
b. contains hydroxyproline
c. depends in part on non-covalent bonds for its strength
d. is a component of basement membranes
e. is formed from procollagen

88. Wound healings is delayed by:
a. insulin
b. ultraviolet light
c. zinc deficiency
d. low temperature
e. glucocorticoids

89. Prostaglandins:
a. are synthesized from endoperoxides
b. have half live in the circulation of 6 hrs
c. are metabolized in the lungs
d. are synthesized in platelets
e. are synthesized by vascular endothelium

90. compare with cow milk, human milk contains more:
a. lactose
b. casein
c. sodium
d. water
e. ascorbic acid

91. The pelvic surface of the sacrum:
a. gives origin to the piriformis muscle
b. gives origin to the levator ani muscle
c. is broader in the male than in the female
d. transmit the dorsal rami of the sacral nerves
e. is in contact with the anal canal

92. Chromaffin cells:
a. are innervated by pre – ganglionic sympathetic nerve fibers
b. are present in the adrenal cortex
c. are derived from neuro- ectoderm
d. can decarboxylate amino acids
e. are present in celiac ganglia

93. in the fetal circulation:
a. the ductus venosus delivers blood directly into the superior vena cava
b. the umbilical artery returns blood from the placenta
c. the ductus aretriosus carries blood to the lungs
d. blood returning from the lungs is 90 % saturated with oxygen
e. blood from the inferior vena cava is largely directed through the foramen ovale

[You must be registered and logged in to see this link.]

94. The following structures take part in the formation of the anterior fontanelle in the fetal skull:
a. lambdoidal suture
b. occipital suture
c. sagittal suture
d. glabella
e. frontal suture
[You must be registered and logged in to see this link.]

95. The ductus venosus:
a. is part of the embryonic heart
b. is a shunt preventing blood from passing to the fetal lungs
c. gives rise to the legamentum teres
d. caries blood with a higher po2 than umbilical blood
e. is derived from the anterior cardinal vien

96. Successful lactation is:
a. maintained by estrogens
b. maintained by progesterone
c. initiated by a prolactin surge
d. maintained by human placental lactogen
e. inhibited by dopamine

97. in a woman of reproductive age, serum concentration of the following hormones exhibit a recognized pattern of diurnal variation:
a. progesterone
b. melatonin
c. cortisol
d. oestrone
e. follicle stimulating hormone

98. Serum concentration of the following increases during pregnancy:
a. sex hormone binding globulin
b. prolactin
c. total thyroxin
d. follicle stimulating hormone
e. 17 alpha – hydroxyprogesterone

99. Luteinzing hormone:
a. is required for normal corpus luteum survival
b. has a half life in the circulation of 30 hrs
c. is released in pulses
d. in the male stimulates testosterone production
e. plasma concentration are increased in post menopausal women

100. The secretion of growth hormone:
a. occurs in the hypothalamus
b. ceases when the adult stage is reached
c. is decreased during stress
d. is increased during fasting
e. is increased with exercise

101. Parathyroid hormone:
a. decrease the renal secretion of phosphate
b. increase calcium resorption in the bone
c. depresses pituitary activity
d. concentration in blood are raised when the calcium level falls
e. increases renal tubular resorption of calcium

102. Aldosterone:
a. reduce sodium resorption in the proximal convoluted tubules
b. reduces sodium absorption in the descending loop of Henle
c. increase sodium absorption in the distal convoluted tubules
d. increase potassium loss from the tubule
e. increase sodium absorption in the collecting tubule

103. The germination of tetanus spores in a wound is inhibited by :
a. tissue trauma
b. oxygen
c. injection of toxin
d. injection of toxoid
e. removal of devitalized tissue

104. cytomegalovirus :
a. is adenovirus
b. may be cultured readily in cell free media
c. is a cause of cerebral calcification
d. causes haemolytic anaemia in the neonate
e. may be transmitted in the saliva

105. hypokalaemia may be caused by :
a. bendrofluazide
b. digoxin
c. spironolactone
d. carbenoxolone
[You must be registered and logged in to see this link.]

e. amiloride

106. The following are required for haemostatic clot formation:
a. conversion of prothrombin to thrombin
b. platelet phospholipids
c. active conversion of plasminogen to plasmin
d. fibrin degradation product
e. antithrombin

107. Myometrial contractility:
a. is calcium dependant
b. is associated with phosphorylation of myosin light chain
c. is independent of cyclic adenosine monophosphate (cAMP)
d. is mediated by somatic nerves
e. depend on myometrial gap junctions

108. The following values fall within the normal range for the adult female bladder:
a. residual urine of 100 ml
b. voiding volume of 250 ml
c. bladder capacity of 900 ml
d. intravesical pressure rise to less than 10 cm H2O during early filling
e. maximum urine flow rate of 60 ml / second

109. in the testis:
a. maturation from spermatogonia to spermatozoa takes about 29 days
b. sertoli cells can mature into spermatids
c. leydig cells produce inhibin
d. luteinzing hormone inhibits the secretion of testosterone
e. large quantities of fructose are present in the seminal fluid

110. Function of the spleen in the healthy adult includes:
a. erythropoiesis
b. destruction of erythrocytes
c. formation of B lymphocytes
d. phagocytosis of bacteria
e. production erythropoietin


[You must be registered and logged in to see this link.]

111. metronidazole :
a. is effective against giardia lamblia
b. is effective when administered by rectum
c. should not be administered intravenously
d. is usually effective against Entamoeba histolytica
e. interfere with ethanol metabolism


[You must be registered and logged in to see this link.]

112. carcinoma in situ in epithelium ( intraepithelial neoplasia) is characterized by :
a. increased mitotic activity
b. loss of polarity
c. increased adhesiveness to underlying stroma
d. pyknosis
e. increased thickness of epithelium

113. the following provide conclusive evidence of pregnancy in uterine curetting :
a. deciduas compacta
b. arias-stella changes in endometrial glands
c. spiral arterioles
d. plasma cell infiltration
e. chorionic villi

114. Complications of myocardial infarction include:
a. fibrous pericarditis
b. aortic aneurysm
c. ventricular mural thrombi
d. coronary atherosclerosis
e. ventricular aneurysm

115. in the pathogenesis of thrombi :
a. prostacyclin induces platelet aggregation
b. platelets synthesized thromboxane A2
c. thromboxane A2 induces vasoconstriction
d. contact with subendothelial collagen causes platelet


116. the characteristic features of addisonian pernicious anaemia include :
a. leucocytosis
b. inheritance as an autosomal dominant trait
c. a raised corpuscular haemoglobin concentration
d. an increased incidence of gastric neoplasia
e. an increased incidence of primary hypothyroidism

117. immunodeficiency states may be associated with :
a. viral infection of T lymphocytes
b. B cell lymphomas
c. glucocorticoids administration
d. haemolytic disease of the new born
e. untreated Hodgkins lymphoma
[You must be registered and logged in to see this link.]


118. concerning immunoglobulins :
a. IgG contain tow heavy chains
b. IgM is produced before IgG in the immune response
c. IgE is the principle immunoglobulin secreted by mucous membranes
d. IgA is the principle immunoglobulin involved in allergic reactions
e. IgA is secreted in breast milk

119. antibodies :
a. are proteins
b. are formed in the fetus before 12 weeks of intrauterine life
c. have an average molecular weight of around 10,000 Daltons
d. of rhesus type are genetically transmitted
e. are produced by the ribosomes of plasma cells

120. the following disorders have an X linked pattern of inheritance :
a. glucose-6-phosphate dehydrogenase deficiency
b. kleinfelters syndrome
c. adrenogenital syndrome
d. haemophilia B
e. familial hypercholesterolemia

121. the following genetic disorders are inherited as autosomal recessives :
a. Duchene muscular dystrophy
b. Huntingdon’s chorea
c. tay-sachs disease
d. retinoblastoma
e. achondroplasia

122. concerning radiation physics :
a. an electron has a greater mass than a proton
b. a positron has the same charge as an electron
c. a proton has a positive charge
d. a neutron has almost the same mass as a proton
e. the hydrogen nucleus is a neutron

123. the conversion of glucose to lactic acid :
a. occur in a single enzymatic reaction
b. is the only pathway for the synthesis of ATP in the red blood cell
c. is a reversible process in skeletal muscle
d. is inhibited by high cellular concentration of ATP
e. occurs in skeletal muscle when the availability of oxygen is limited

124. normal human seminal fluid :
a. coagulates in vitro
b. conatins sucrose
c. has a pH of 5
d. may contain 15 % of morphologically abnormal spermatozoa
e. is predominantly produced within the testis

125. glucocorticoids :
a. promotes hepatic gluconeogenesis
b. suppress uptake of glucose by the muscle
c. promote protein breakdown
d. promote fat breakdown
e. increase glycolysis in adipose tissue

126. cholesterol :
a.Is essential dietary requirement
b. is present in the plasma membrane of all human cells
c. can not be synthesized by the liver
d. is transported in the circulation bound to albumin
e. is a precursor for the synthesis of steroid hormones .

127. fetal haemoglobin (HbF) :
a. is not formed before 20 weeks of intrauterine life
b. is more resistant than adult haemoglobin to denaturation by alkali
c. in the fetus constitutes 80-90 % of the haemoglobin at term
d. represent less than 5 % of total haemoglobin 8 weeks after birth
e. is increased in adult patients with beta thalassaemia

128. glucagon :
a. is a polypeptide hormone
b. is secreted by beta cells of the pancreatic islets
c. causes muscle glycogenolysis
d. has a half life of 5-10 minutes in the circulation
e. secretion is stimulated by cortisol

129. the female breast :
a. has a base which extend to the ninth rib
b. consist of up to 10 lobes
c. partially overlies the latissmus dorsi muscle
d. receives blood from the internal thoracic artery
e. is supplied in part by supraclavicular nerves

130. labia majora :
a. have a subcutaneous layer of non-striated muscle
b. have a fibro-fatty cord
c. contain the termination of the round ligament of the uterus
d. unite anteriorly forming the fourchette
e. have the openings of the greater vestibular glands on their medial surfaces .

131. the external dural (epidural) space :
a. contains the vertebral venous plexus
b. lie between pia and dura mater
c. contains no cerebrospinal fluid
d. ends at the level of the second lumbar vertebra
e. begins at the level of the first cervical vertebra

132. the following become occluded at or shortly after birth :
a. hepatic veins
b. ductus venosus
c. sinus venosus
d. ductus arteriosus
e. proximal ends of the hypogastric arteries

133. in the cell cycle :
a. DNA synthesis takes place during the M (mitosis) phase
b. RNA synthesis takes place predominantly during the S (synthesis ) phase
c. protein synthesis occurs during the G1 (pre-synthesis ) phase
d. microtubule assembly takes place during the G2 (post-synthesis ) phase
e. chromatids migrate to poles during metaphase

134. the peritoneum :
a. is lined by a stratified squamous epithelium
b. has a visceral layer united firmly with underlying organs
c. has a parietal layer innervated by somatic spinal nerves
d. has venous drainage to the portal vein
e. can function as a dialysing membrane

135. the following are clinical manifestation of infection of Coxsackie’s viruses :
a. aseptic meningitis
b. endometritis
c. epidemic myalgia
d. herpangina
e. pericarditis

136. prolactin secretion :
a. increases during sleep
b. increases during exercise
c. is increased by thyrotropin-realising hormone (TRH)
d. is decreased by dopamine
e. is increased by estrogen

137. in statistics a normal distribution :
a. has 50 % of its values on or above the mean
b. has 95 % of its values within tow standard deviations of the mean
c. has 10 % of its values below the 90th centile
d. has all its values below the third quartile
e. has 99 % of its values within three standard deviations of the mean

138. after the menopause :
a. calcium excretion decreases
b. there is a decreased secretion of follicle stimulating hormone
c. there is an increased secretion of progesterone
d. the vagina pH is increased
e. the endometrium becomes unresponsive to the action of estrogen

139. in the human female :
a. no new ova are formed after birth
b. miotic division is completed by puberty
c. prostaglandins are produced in the endometrium
d. average menstrual blood loss is about 150 ml
e. menstrual flow is maintained from the basal arteries


[You must be registered and logged in to see this link.]

140. the following viruses are commonly transmitted by coitus :
a. hepatitis B virus
b. human papilloma virus
c. herpes simplex virus
d. ebstein bar virus
e. cytomegalovirus

141. therapeutic substances predominantly metabolized by the liver include :
a. gentamicin
b. frusemide
c. morphine
d. phenothiazine
e. chlorpropamide

142. the following drugs are frequently used in the treatment of the named infections :
a. co-trimoxazole __ vaginal candidiasis
b. ampicillin – tuberculosis
c. tetracycline – trachoma
d. chloramphenicol – purulent conjunctivitis
e. trimethoprim – trichomoniasis

143. concerning haemoglobin :
a. haem contain ferric iron
b. the alpha globulin chain contains more than 100 amino acids
c. the affinity of haemoglobin for oxygen is modified by the red blood cell concentration of 2-3 , diphosphoglycerate
d. the oxygen affinity of deoxyhaemoglobin is greater than that of haemoglobin
e. a fall in pH shifts the haemoglobin oxygen dissociation curve to the left

144. Hereditary spherocytosis:
a. is inherited as a sex linked recessive characteristic
b. is associated with a deficiency of red cell pyrineratekinase
c. is associated with decreased fragility of red cells
d. may present in the first month of life
e. is associated with splenomegaly

145. stimulation of alpha adrenergic receptors :
a. does not occur with nor adrenaline
b. in the blood vessels of the skin leads to vasoconstriction
c. increases intestinal peristalsis
d. leads to contraction of erector pili muscles
e. leads to constriction of the pupil

146. epithelioid cells :
a. are transformed tissue macrophages
b. are found in Crohns disease
c. are found in granulation tissue
d. contains phagosomes (secondary lysosomes )
e. may undergo mitosis

147. in chronic infectious disease :
a. there is hyperplasia of the reticulo – endothelial system
b. specific IgG production is diminished
c. humorally mediated hypersensitivity reactions are common
d. anaemia is often of microcytic type
e. serum albumin levels are reduced

148. the following ions are predominantly extracelluar :
a. sodium
b. chloride
c. magnesium
d. phosphate
e. potassium

149. disease transmitted by insect vector :
a. trypansomiasis
b. yellow fever
c. achistosomiasis
d. leishmaniasis
e. filariasis

160. penicillin :
a. may be active against gram negative bacteria
b. interferes with normal cell wall synthesis
c. is inactivated by beta lactamase enzymes
d. is effective against resting bacterial cells
e. is contraindicated in pregnancy

161. the fetal testis :
a. are essential for the formation of the mesonephric (Wolffian) ducts
b. are essential of the formation of the paramesonephric (Mullerian)ducts
c. secretes testosterone
d. arise from the gonadal (genital) ridge
e. differentiate in response to a product of the Y chromosome

162. tetany may occur as a complication of :
a. osteoporosis
b. neonatal hypocalcaemia
c. respiratory alkalosis
d. peripheral neuropathy
e. untreated hyperparathyroidism

163. the tidal volume :
a. is about 700 ml at rest in pregnancy
b. is twice the anatomical dead space
c. together with the residual volume equals the vital capacity
d. is less than alveolar volume
e. is decreased by estrogens

164. the release of catecholamine from the adrenal medulla increases:
a. during sleep
b. when the nerves to the adrenal gland are stimulated
c. when the blood sugar rises
d. during excitement
e. in the presence of a phaeochromocytoma

165. the pineal gland :
a. is stimulated at the anterior end of the third ventricle
b. is innervated by the parasympathetic nervous system
c. produces melatonin
d. may be calcified in the adult
e. is most active during daylight

166. saliva :
a. contain an enzyme which is essential for the complete digestion of stomach
b. contains secretor substances
c. keeps the pH in the mouth between 5.0 and 6.0
d. has an antibacterial action
e. contains mucin

[You must be registered and logged in to see this link.]

167. the secretion of ant6idiuretic hormone is :
a. inhibited by ethyl alcohol
b. controlled by nervous stimuli from the hypothalamus
c. increased by a rising plasma osmolality
d. decreased by a falling plasma volume
e. inhibited by distension of the right atrium

168. mRNA :
a. is a double stranded polymer
b. is transcribed from DNA in the nucleus
c. is not present in reticulocytes
d. contains adenosine
e. is not present in oocytes

169. hypotension causes :
a. diminished urine production
b. inhibition of rennin secretion
c. stimulation of aldosterone secretion
d. stimulation of angiotensin II production
e. increased excretion of sodium

170. in the human the haploid number of chromosomes is found in the :
a. red blood cells
b. blastocyte
c. primary oocytes
d. 1st polar body
e. spermatozoon

171. the following statements relate to plasma proteins :
a. they create an oncotic pressure of 3.3 KPa (25 mm Hg ) across capillary walls
b. they form a major part of the plasma cationic pool
c. all are manufactured in the liver
d. albumin has a lower molecular weight than fibrinogen
e. fibrinogen is freely filtered at the glomerulus

172. in a healthy woman , rennin :
a. is secreted only by the kidney
b. plasma concentration is greater in the pregnant than in the non pregnant state
c. plasma concentration is increased by diuretic therapy
d. converts angiotensinogen into angiotensin II
e. activity is blocked by captopril

173. concerning human parturition :
a. the number of oxytocin receptors in the myometrium increases before the onset of labour
b. in the primigravida , cervical dilation usually precedes cervical effacement
c. the plasma oxytocin concentration increases at the onset of labour
d. oxytocin stimulates the synthesis of prostaglandins within the uterus
e. contraction of the maternal abdominal muscles is essential for spontaneous vaginal delivery

174. recognized effects of pregnancy include :
a. transient impairment of glucose tolerance
b. a raised glomerular filtration rate
c. a raised plasma concentration of free tyrosine
d. a reduced plasma concentration of alkaline phosphatase
e. an increased secretion of prolactin

175. during DNA replication :
a. only one strand of DNA is copied f
b. replication is semi conservative t
c. the DNA helix is unwound by the enzyme helicase t
d. DNA is synthesized by DNA synthetase f

176. the following are component of nucleotides :
a. purine or pyrimidine base t
b. pentose sugar t
c. phosphate group t
d. histones f


m
177. during DNA replication :
T a. okazaki fragments are later joined by DNA ligase
T b. DNA polymerase can detect base mis- match and correct any errors in DNA synthesis
F c. replication occurs during the G- phase of the cell cycle
T d. replication is bi-directional

178. with respect to transcription :
F a. the primary mRNA transcript is produced only from the exons on the DNA molecule
T b. introns have a sequence that begins with 5-GU
T c. introns have a sequence that ends with AG-3
T d. the removal of introns occurs in spliceosomes

179. transfer RNA :
F a. makes up to 80 % of total cellular RNA
T b. binds specific amino acids
F c. carries amino acids bind to its 5 end to polyribosomes
T d. has a clover-leaf secondary structure

180. the following are recognized post-translations modifications of proteins :
T a. formation of disulphide bonds
T b. excision off single sequences
T c. addition of oligosaccharides to asparagines residues
T d. proteolysis

181. with respect to protein synthesis from mRNA :
T a. each codon is specific for a particular amino acid
T b. one amino acid can be coded by more than one codon
F c. there are total of 48 possible codons
F d. during the synthesis of aminoacyl-tRNA from amino acids and tRNA , ATP is converted to ADP phosphate

182. during gluconeogenic phase of starvation :
F a. glycolysis is stimulated
T b.amino acids are released from skeletal muscle protein
F c. there is increased fatty acids synthesis
T d. phosphfructose is inactivated

183. during glycolysis :
F a. the initial reaction is the conversion of glucose to pyruvate
F b. glucose is converted to glucose-6- phosphate by glucose phosphatase
F c. the formation of glucose-6-phosphate from glucose requires the hydrolysis of ATP to AMP
T d. the formation of glucose-6-phosphate from glucose + ATP is an irreversible reaction

184. with respect to the regulation of glycogen metabolism :
T a. glycogen synthase is activated by dephosphorylation
T b. glucagon stimulates the phosphorylation of glycogen phosphorylase
T c. adrenaline stimulate the phosphorylation of glycogen synthase
F d. insulin result in the phosphorylation and inactivation of protein phosphatase 1

185. during glycogen breakdown :
F a. skeletal muscle glycogen can be converted to glucose and released into the circulation
T b. hepatic glycogen can be converted into glucose and released into the circulation
F c. the enzyme phosphorylase breaks alpha 1,4 glycosidic bonds less than 4 residues away from a branch point
F d. the enzyme phosphorylase requires the coenzyme biotin

186. with respect to the regulation of glycogen metabolism :
F a. insulin causes the dephosphorylation and activation of phosphorylase
T b. insulin causes the dephosphorylation and activation of glycogen synthase
T c. calcium ions stimulate glycogen breakdown in skeletal muscle
F d. skeletal muscle glycogen can be converted to glucose

187. the following are recognized classes of enzymes :
F a. digestive enzymes
F b. polymerases
T c. oxidoreductases
T d. transferases

188. enzymes :
T a. increase the rate at which a reaction reaches equilibrium
F b. shift the equilibrium point of a reaction towards the synthesis of product
F c. are inactivated at the end of the reaction
F d. increases the equilibrium constant of the reaction catalysed



189. with respect to inhibitors of enzyme action :
T a. irreversible inhibitors cause permanent inhibition of enzyme activity
T b. competitive inhibitors are usually structurally similar to the enzyme substrate
T c. the action of competitive inhibitor may be overcome by increasing substrate concentration
F d. competitive inhibitors are never converted to substrate

190. with respect to the kinetics of enzyme reaction :
T a. the Lineweaver-Burk plot has 1/initial velocity on the y-axis
T b. the sI unit of enzyme activity is the katal
F c. one katal is the quantity of enzyme in the presence of which one mole of substrate is converted per minute under optimal conditions
T d. the standard unit of enzyme activity U is the amount of enzyme catalysing the formation of one micromole of substrate per minute under optimal conditions

191. during the digestion of carbohydrates :
T a. salivary amylase hydrolyses alpha 1,4 glycosidic bonds in starch
F b. pepsin hydrolyses starch to disaccharides in the stomach
F c. pancreatic amylase has optimal enzyme activity at pH <3
F d. sucrase is secreted by the pancreas

192. vitamin B12 :
F a. can be synthesized de novo by humans
F b. is present in plant food
F c. is not stored by humans in significant amount
F d. is destroyed by cooking

193. thiamine :
F a. is not destroyed by cooking
F b. deficiency causes scurvy
T c. deficiency occurs in chronic alcoholics
T d. deficiency occurs in hyperemesis gravidarum

194. vitamin B12 deficiency occurs :
T a. in vegans
T b. following gastrectomy
F c. following biliary obstruction
T d. in celiac disease


195. vitamin D :
F a. metabolism by the kidney is principally regulated by calcitonin
F b. has its action through a membrane bound receptor
F c. deficiency causes renal failure
F d. deficiency causes osteoporosis

196. vitamin B12 :
T a. deficiency result in impaired DNA synthesis
T b. is a co-enzyme in the metabolism of metheyltetrahydrofolate
T c. requires intrinsic factor for it is absorption in the gut
F d. is transported bound to cyanocobalamine

197. necrosis :
F a. occurs when tissues are rapidly frozen
F b. is characterized by cytoplasmic basophilia
T c. may be associated by karyolyasis
T d. may be associated with karyorrhexis

198. dyspalsia :
T a. can be graded
T b. when severe represent carcinoma in situ
F c. does not occurs in the cervix
T d. can occur in the stomach

199. with respect to tissue calcification :
F a. metastatic calcification only occurs in malignant tissue
T b. metastatic calcification occurs in hyperparathyroidism
F c. metastatic calcification only occurs in damaged tissues
T d. metastatic calcification is associated with multiple myeloma

200. atrophy :
T a. is a decrease in the size of an organ during post natal life
T b. can occurs in the skeletal muscle
F c. is always physiological
F d. is associated with an increase in cell number

201. the following are characteristics of benign tumours :
F a. invasion into surrounding structures
T b. well defined capsules
F c. large size
F d. marked pleomorphism of the tumour cells


202. with respect to cervical intraepithelial neoplasia(CIN) :
F a. CIN I is characterised by the presence of dysplastic cells in the upper third of the epithelium
T b. CIN III is characterised by the presence of dysplastic cells throughout the epithelial layer
T c. CIN I may regress spontaneously
F d. CIN II is characterised by the presence of dysplastic cells in the upper tow thirds of the epitheliu


203. Cervical ectropion:
F a. is associated with dysplasia
F b. is caused by inversion of the distal endocervical canal
T c. is associated with use of the combined oral contraceptive pill
F d. is associated with human papilloma virus infection

204. Clear cell carcinoma of the vagina:
T a. is an adenocarcinoma
T b. is associated with vaginal adenosis
F c. typically present in post menopausal women
T d. is associated with in-utero exposure to diethylstilboestrol

205. Vulval intra-epithelial neoplasia:
T a. is premalignant
F b. is characterised by decreased nuclear cytoplasmic ratio in epithelial cells
T c. is associated with human papilloma virus infection
F d. is classified as VIN III if atypical cells occupy the lower third of the epithelium only

206. The following organs are typically involved in the direct invasion of cervical carcinoma:
T a. the ureter
T b. the bladder
F c. the internal iliac nodes
T d. the lower third of the vagina

207. Atypical endometrial hyperplasia:
F a. can readily be differentiated histologically from well differentiated endometrial carcinoma
F b. typically present around the menarche
F c. does not pro0gress into endometrial carcinoma
T d. typically present with peri – menopausal bleeding

208. uterine leiomyomas :
T a. typically reduce in size during menopause
F b. do not increase in size during pregnancy
F c. are pre malignant lesions
T d. are recognised cause of menorrhagia

209. adenomyosis :
T a. typically present with dysmenorrhoea
F b. is most common during menarche
T c. is the presence of endometrial tissue within the myometrium
F d. can be diagnosed at laparoscopy

210. chronic endometritis :
T a. is associated with infection with mycobacterium tuberculosis
T b. is characterised by lymphocytes and plasma cell infiltration
T c. can occur following miscarriage
T d. is associated with the intra – uterine contraceptive device

211. atypical endometrial hyperplasia :
F a. occurs as a progression of simple hyperplasia
T b. can occur in the absence of simple typerplasia
T c. is typically focal rather than diffuse
F d. is associated with stromal proliferation

212. the following are characteristic features of polycystic ovary syndrome :
F a. the presence of multiple 5 cm cysts in the ovary
T b. thickening of the ovarian capsule
T c. ovarian stromal hyperplasia
T d. leutenisation of the theca cells

213. carcino-embryonic antigen :
T a. is more likely to be elevated in women with mucinous compared to serous ovarian cancers
T b. is more likely to be activated in undifferentiated compared to well differentiated tumours
F c. is elevated in women with endometriosis
F d. is elevated in women with large uterine fibroids

214. CA – 125 levels are typically elevated in the following :
F a. ovarian terratomas
T b. ovarian hyperstimulation syndrome
T c. endometroid adenocarcinoma of the ovary
T d. pelvic inflammatory disease

215. with respect to hereditary ovarian cancer :
F a. 1% of ovarian cancers are hereditary
F b. the BRCA-1 mutation is inherited in an autosomal co-dominant fashion
F c. the prognosis of ovarian cancer is better in women with the BRCA-1 mutation than in those with sporadic cancers
T d. the BRCA-1 mutation is associated with an increased risk of colon cancer in males

216. ovarian thecomas :
T a. may be associated with pleural effusion
T b. are benign tumour
F c. secrete progesterone
F d. are cystic tumours

217. polymorphonuclear neutrophils :
F a. differentiate into monocytes
T b. are peroxidase positive
T c. are alkaline phosphatase positive
T d. contain collagen

218. tumour necrosis factor – alpha :
T a. is secreted by macrophages
F b. is secreted by neutrophils
T c. is secreted by monocytes
T d. induce cachexia

219. with respect to the complement system :
T a. the lectin pathway is a component of the innate immune system
T b. the lectin pathway is activated by the binding of mannose-binding lectin to the surface glycogen on microbes
T c. phagocytes express receptors for C3b
T d. C3b causes opsonization of bacteria

220. natural killer cells :
T a. do not express T- cell receptors
T b. are activated by interleukin-12
T c. are granular cells
T d. induce apoptosis in virally infected cells


221. polymorphonuclear neutrophils :
F a. stain with acid dyes
F b. stain with basic dyes
F c. divide by mitosis within the circulation
F d. are multi-nucleated

222. antibodies :
T a. serves as B-cell antigen receptors
T b. may be membrane-bound
T c. are present in breast milk
F d. do not cross the placenta

223. with respect to the structure of antibodies:
F a. the Fc fragment includes part of the light chain
T b. each antibody molecule has tow identical Fab regions
F c. each antibody molecule has tow Fc regions
T d. the hinge region is located between the Fab and Fc region


224. with respect to the peritoneal ligaments :
F a. the lateral umbilical ligament extend from the external iliac artery to the umbilicus
T b. the lateral umbilical ligament is obliterated umbilical artery
F c. the lienorenal ligament contains the renal artery
F d. the gastrosplenic ligament connects the lesser curvature of the stomach to the spleen

225. with respect to the duodenum :
F a. the head of the pancreas lies inferior to the third part of the duodenum
F b. the ligament of Treitz is attached to the third part of the duodenum
T c. the fourth part of the duodenum is related posteriorly to the aorta and left psoas muscle
T d. the third part of the duodenum is related posteriorly to the right ureter and right psoas muscle

225. with respect to the biliary tree :
T a. the bile canliculi drain into the interlobular ducts
F b. the interlobular ducts are located at the centre of the hepatic lobule
F c. the right hepatic duct drains bile from the caudate lobe of the liver
T d. the left hepatic duct drains bile from the quadrate lobe of the liver

226. the greater omentum :
T a. has its anterior layer attached to the greater curvature of the stomach
T b. has its posterior layer attached to the inferior border of the transverse colon
T c. has the lesser sac between the anterior and posterior layers
T d. contains the right gastro-epiploic artery

227. the cecum :
F a. is retroperitoneal
F b. has three bands of longitudinal muscles called appendices epiploicae
F c. has an outer continuous layer of longitudinal muscle
T d. does not have a mesentery


228. with respect to the surface anatomy of the abdomen :
T a. the sub –costal plane joins the lowest points of the costal margin on each side
F b. the sub –costal plane is at the level of the body of the 5th lumbar vertebra
F c. the inter-tubercular plane joins the ischial tuberosities on each side
T d. the inter-tubercular plane is at the level of the body of the 5th lumbar vertebra

229. with respect to the muscles of the posterior abdominal wall :
F a. psoas major inserts onto the greater trochanter
T b. iliacus inserts onto the lesser trochanter
T c. iliacus is supplied by the femoral nerve
T d. iliacus flexes the thigh on the trunk

230. with respect to the scrotum and its contents :
T a. dartos muscle is smooth muscle
F b. the external spermatic fascia lies superficial to dartos muscle
T c. cremasteric fascia lies deep to the external spermatic fascia
F d. the tunica vaginalis cover the anterior , posterior and lateral aspects of the testis

231. with respect to the scrotum and its contents :
F a. the lymphatic drainage of the scrotum is to the deep inguinal nodes
F b. lymphatic drainage of the testis is to the Para-aortic nodes at the level of L5
F c. the temperature in the scrotum is maintained at 3C above body temperature
F d. the pampiniform plexus is formed by the testicular artery

232. the following structures are present within the inguinal canal in the male :
T a. testicular artery
T b. vas deferens
F c. inguinal nerve
T d. genital branch of the genitor-femoral nerve

[You must be registered and logged in to see this link.]

233. the portal vein :
F a. is formed by the union of the superior and inferior mesenteric veins
F b. lies anterior to the first part of the duodenum
T c. lies within the lesser omentum
F d. lies posterior to the opening into the lesser sac

234. with respect to the blood supply to the gastrointestinal tract :
F a. the inferior pancreatico-dudenal artery is a branch of the celiac artery
F b. the middle colic artery is a branch of the inferior mesenteric artery
T c. the ileocolic artery is a branch of the superior mesenteric artery
T d. the inferior mesenteric artery is the artery to the hind gut

235.with respect to the lumbar plexus :
T a. the genitor –femoral nerve derived from L1 & L2
T b. the genitor-femoral nerve emerges from the lumbar plexus on the anterior surface of psoas major
T c. the genital branch of the genitor-femoral nerve enters the spermatic cord
F d. the femoral branch of the genitor –femoral nerve supplies the cremasteric muscle

236.the inferior vena cava within the abdomen :
T a. lies to the right of the aorta
F b. pierces the central tendon of the diaphragm at the level of T12
T c. lies medial to the right ureter
T d. is related anteriorly to the opening into the lesser sac

237.the celiac artery :
F a. is the artery of the mid gut
F b. lies anterior to the lesser sac
T c. is a branch of the abdominal aorta at the level of the 12th thoracic vertebra
F d. has three branches – the right gastric artery, the splenic artery and the hepatic artery

238. with respect to the sacral plexus :
F a. the sciatic nerve is formed from S 1,2&3
T b. the sciatic nerve leave the pelvis through the greater sciatic foramen
T c. the superior gluteal nerve is the largest nerve in the body
F d. the superior gluteal nerve leaves the pelvis through the lesser sciatic foramen

239.the levator ani muscle :
T a. originates from the back of the pubic bone anteriorly
T b. originates from the obturator fascia laterally
T c. originates from the ischial spine
F d. the anterior fibres are inserted onto the symphysis pubis

240. with respect to the hypogastric plexuses :
F a. the superior hypogastric plexus lies in the retro-peritoneal tissue between the tow internal iliac arteries
T b. the superior hypogastric plexus lies anteriorly
T c. . the superior hypogastric plexus contains both sympathetic and Para sympathetic fibres
F d. . the superior hypogastric plexus receives branches from the pelvic aplanchnic nerves

241. with respect to the sacrum :
T a. the sacral canal is formed by the vertebral foramina of the sacral vertebrae
F b. the sacral canal contains the spinal cord
T c. the sacral canal contains the filum terminale as far down as the level of S5
T d. the sacral canal contains the cauda equine as far down as the level of S2

242. the pelvic splanchnic nerves :
T a. join the inferior hypogastric plexus directly
F b. join the superior hypogastric plexus directly
T c. provide para sympathetic supply to the hind gut
F d. provide sympathetic supply to the bladder

243. the sigmoid colon :
T a. receives its nerve supply from the inferior hypogastric plexuses
F b. has an outer continuous layer of longitudinal smooth muscle
T c. lymphatic drainage is to the inferior mesenteric nodes
F d. is supplied by the sigmoid branches of the internal iliac artery

244. the cervix :
F a. is supplied by the cervical artery , a branch of the internal iliac artery
F b. receives nerve supply from the pudendal nerve
T c. undergoes cyclical changes during the menstrual cycle
F d. sheds its lining during menstruation
245. the following normal structures can be palpated through the vagina :
F a. the ureters
T b. the ovaries
F c. the fallopian tubes
T d. the uterus

246. the uterine artery :
F a. is a branch of the posterior division of the internal iliac artery
F b. is crossed by the ureter
F c. runs medially in the infudibulo- pelvic ligament
F d. reaches the pelvis at the level of the external os

247. with respect to the rectum :
F a. The middle third is covered by peritoneum on its anterior and lateral surfaces
F b. the lower third is covered by peritoneum on its anterior surface only
T c. the lumen has tow transverse folds on the left and one on the right rectal wall
F d. lymhatics from the lower third of the rectum drain into the external iliac nodes

248. the superficial perineal pouch in the female :
T a. contains the bulb of the vestibule
F b. lies between the tow layers of the urogenital diaphragm
T c. contains the perineal branch of the pudendal nerve
T d. contains branches of the internal pudendal artery

249. the bulbospongiosus muscle :
F a. surround the orifice of the urethra
T b. surround the orifice of the vagina
T c. covers the bulb of the vestibule
F d. is attached to the crus of the clitoris

250. the superficial transverse perineal muscle :
F a. lies in the anterior part of the superficial perineal pouch
F b. originates from the ischial tuberosity
T c. is inserted onto the perineal body
T d. is supplied by the perineal branch of the pudendal nerve



251. the pudendal nerve :
T a. supply the mucus membrane of the lower half of the anal canal
T b. supplies the perineal skin
T c. supplies the clitoris
F d. gives off the superior rectal nerve

252. with respect to the external genitalia in the female :
T a. the vestibule is triangular in shape
F b. the vestibule is bounded anteriorly by the perineal body
F c. the opening of the urethra lies anterior to the clitoris
F d. the vestibule is bounded laterally by the labia majora


253. the rib :
T a. has a head which articulate with a vertebral body
T b. has a tubercle which articulates with the transverse processes of the numerically corresponding vertebra
F c. has an angle between the head and the tubercle
F d. has a neck between the tubercle and the angle

254. the diaphragm :
F a. has a central muscular part and a peripheral tendinous part
F b. has a sternal origin from the p[posterior surface of the body of the sternum
F c. has a costal origin from the deep surfaces of the lower 4 ribs and their costal cartilages
F d. has a right crus originating from the medial arcuate ligaments

255. the mammary glands :
T a. receives arterial supply from the internal thoracic artery
T b. receives arterial supply from the axillary artery
T c. receives arterial supply from the inter costal arteries
F d. divided into 5-10 lobes radiating from the nipple

256. the diaphragm :
F a. is covered by visceral pleura on its superior surface
T b. receives motor nerves supply from the phrenic nerve
T c. receives sensory nerves supply from the phrenic nerve
T d. receives sensory supply from the lower 5 inter costal nerves



257. The first rib :
T a. articulate with the first thoracic vertebra
T b. articulates with the body of the 7th cervical vertebra
T c. is crossed by the subcalvian vein anterior to the scalene tubercle
F d. is crossed by the subcalvian artery posterior to the scalene tubercle

258. the right ventricle :
F a. has a pulmonary out flow tract (pulmonary valve ) with tow semilunar caps
F b. contracts during ventricular diastole
F c. communicates directly with the inferior vena cava
F d. pumps oxygenated blood into the pulmonary artery

259. with respect to the surface markings of the heart valves :
T a. the tricuspid valve lies behind the right half of the sternum opposite the 4th inter costal space
T b. the mitral valve lies behind the left half of the sternum opposite the 4th inter costal cartilage
T c. the pulmonary valve lies behind the medial end of the third left costal cartilage
T d. the aortic valve lies behind the left half of the sternum opposite the third intercostal space

260. with respect to the conducting system of the heart :
T a. the Sino atrial nodes is made of specialized cardiac muscle
F b. the Sino atrial nodes is located within the wall of the left atrium
T c. the atrio ventricular nodes is located in the lower part of the atrial septum
F d. the Sino atrial nodes is connected to the atrio ventricular node by the atrio ventricular bundle

261. the left ventricle :
F a. pumps deoxygenated blood to the lungs
F b. communicates with the right ventricle
F c. communicates with the aorta via the aortic valve which has tow semilunar cusps
T d. communicates with the left atrium via mitral valve which has tow cusps


262. the left ventricle :
T a. has trabeculae carneae
F b. has moderator band
F c. has three papillary muscles
T d. has a thicker wall than that of the right ventricle


263. the following are derivatives of neural crest :
F a. sensory ganglia of all the cranial nerves
T b. melanocytes
T c. the cartilage of the pharyngeal arches
T d. schwann cells

264. the primitive streak :
F a. appears in the surface of the hypoblast on day 15-16
F b. has the primitive node located at its caudal end
F c. formation leads to gastrulation and the formation of the extra-embryonic mesoderm
T d. formation leads to gastrulation and the formation of the intra- embryonic endoderm

265. with respect to the process of oogenesis :
F a. the zona pellucida develops after ovulation
F b. the first meiotic division is completed before puberty
F c. the second meiotic divisions is completed before ovulation
T d. the second meiotic division is completed only if the oocyte is fertilized

266. fertilization :
T a. occurs in the ampulla of the fallopian tube
F b. results in completion of the first meiotic division in the oocyte
F c. results in the cortical granules of the oocyte being released into the cytoplasm
T d. occurs before the completion of the second meiotic division in the oocyte

267. the following are derivatives of the neural crest :
T a. dorsal root ganglia
T b. pre aortic sympathetic ganglia
T c. the sympathetic chain
T d. parasympathetic ganglia
















Last edited by mandible on Mon Sep 06, 2010 6:36 pm; edited 9 times in total
mandible
mandible
جني اصلي
جني اصلي

عدد المساهمات : 600
نقاط : 1800
السٌّمعَة : 9
تاريخ التسجيل : 2010-02-24

بطاقة الشخصية
gini: 4

https://mandible.yoo7.com

Back to top Go down

MCQS Part 1 MRCOG More than 300 stem Empty Re: MCQS Part 1 MRCOG More than 300 stem

Post by dr-dina Mon Aug 02, 2010 4:34 am

تسلم دكتور

بس بعد إذنك لو فى اجابات ياريت حضرتك تحطهالنا
dr-dina
dr-dina
جني مبتدي
جني مبتدي

عدد المساهمات : 3
نقاط : 8
السٌّمعَة : 8
تاريخ التسجيل : 2010-07-11
الموقع : www.egymedicine.net

Back to top Go down

MCQS Part 1 MRCOG More than 300 stem Empty Re: MCQS Part 1 MRCOG More than 300 stem

Post by mandible Sat Aug 28, 2010 3:10 pm

268. during the development of the gastrointestinal system:
F a. the dorsal pancreatic bud gives rise to the uncinate process of the pancreas
F b. the pancreas secretes insulin from 12 weeks gestation
F c. A Meckels diverticulum is remnant of the alantois
F d. the spleen develops from a diverticulum of the foregut

269. during the development of the gastrointestinal system:
T a. the foregut and hind gut are initially closed from the amniotic cavity by the bucco- pharyngeal and cloacal membranes
F b. the mid gut is connected to the yolk sac by the alantois
F c. the smooth muscle of the gut is derived from endoderm
T d. the trachea and bronchi develop as a diverticulum of the fore gut

270. with respect to the development of the urinary tract :
F a. there is an increased number of nephrons in each kidney after birth
F b. the mesonephric kidney develops in the pelvis
T c. the mesonephric kidney develops in the pelvis and ascends into the abdomen
F d. the ureters are endodermal in origin

271. with respect to the circulatory system in the fetus :
F a. blood flows from the left atrium into the right atrium through the foramen ovale
T b. the resistance within the pulmonary vessels is high
T c. oxygenated blood from the placenta is mixed with deoxygenated blood within the right atrium
T d. oxygenated blood from the placenta is mixed with deoxygenated blood in the inferior vena cava

272. with respect to the circulatory system in the fetus :
F a. the obliterated umbilical vein formed the ligamentum venosum
T b. the obliterated ductus venosus formed the ligamentum venosum
T c. the ductus venosus connects the umbilical vein to the inferior vena cava
F d. the ductus arteriosus connects the pulmonary vein to the aorta

273. the human placenta :
F a. has negligible oxygen requirement
T b. has syncytiotrophoblast as the major component of trophoblast at term
T c. is haemochorial
F d. has anastomoses between villous vessels

274. with respect to the transfer of water between the mother and the fetus :
F a. 3-4L of water are exchanged between the maternal and fetal compartment per 24h
T b. water crosses the placenta by osmosis
F c. net water accumulation by the fetus stops at 37 weeks gestation
F d. maternal dehydration has no impact on net fetal water accumulation

275. with respect to the transfer of solutes across the placenta :
F a. glucose transfer is by simple diffusion
F b. amino acid transfer is by active transport
F c. amino acid concentrations are lower in the fetal than in maternal plasma
T d. glucose concentration is lower in the fetal than in maternal plasma

276. trophoblast :
T a. develops from the blastocyst
T b. secretes gondotrophins
T c. is physiologically invasive
T d. forms part of the chorion

277. in the human placenta :
T a. intervillous space communicate directly with the uterine arteries
T b. decidual cells are derived from endometrial stromal cells
F c. at term , each lobe represents a single primary stem villus
F d. cytotrophoblast are in direct contact with maternal blood

278. the corpus luteum :
F a. is formed mainly from the cells of theca interna
T b. secretes progesterone
T c. secretes estrogen
F d. is only formed if fertilization occurs
279. human placental lactogen :
T a. is secreted by the trophoblast
F b. is a glycoprotein
F c. is an insulin agonist
F d. inhibits lypolysis

280. estrogen
T a. inhibits milk secretion by the breast
T b stimulates osteobalstic activity
F c. delays fusion of the epiphyses of long bones
T d. stimulates fat deposition in sub-cutaneous tissue

[You must be registered and logged in to see this link.]

281. sertoli cells :
F a. secretes testosterone
T b. secretes inhibin
T c. secrete Mullerian inhibitory factor
T d. secrete estradiol produced from testosterone

282. Progesterone secretion by the placenta:
T a. reduces myometrial sensitivity to oxytocin
F b. stimulates prostaglandin synthesis in the myometrium
F c. decreases respiratory rate
T d. stimulates prolactin production in the decidua

283. thyroid stimulating hormone :
F a. secretion is stimulated by somatostatin from the hypothalamus
F b. secretion is stimulated by cortisol
T c. secretion is inhibited by growth hormone
F d. inhibits protein synthesis in thyroid gland

284. thyrotropin releasing hormone :
T a. is a tripeptide
F b. is produced by the anterior pituitary gland
T c. stimulates prolactin production
F d. is a glycoprotein

285. Adrenocorticotropic hormone ( ACTH) :
T a. is secreted in conjunction with beta endorphin
T b. stimulates adrenal androgen secretion
F c. has effect on adrenal aldosterone secretion
F d. binds to a receptor located within the cell nucleus

286. antidiuretic hormone :
T a. secretion is stimulated by dehydration
F b. causes a decrease in a urine osmolarity
F c. causes an increase in plasma osmolarity
T d. decreases water excretion by the kidneys

287. thyroid stimulating hormones :
T a. causes hyperplasia and hypertrophy of the follicular cells of the thyroid gland
F b. inhibits proteolysis of thyroglobulin
T c. secretion is inhibited by thyroxine
T d. secretion is inhibited by dopamine from the hypothalamus

288. parathyroid hormone :
F a. acts through a receptor located within the cytoplasm
F b. inhibits the synthesis of 1,25-dihydroxy D3 in the kidneys
F c. increases renal calcium excretion
F d. increase renal resorption of sodium and potassium

289. thyroxine :
T a. stimulates oxygen consumption
F b. decrease cardiac output
F c. increase plasma cholesterol concentration
T d. stimulates cartilage growth

290. with respect to vit D synthesis and metabolism :
F a. Cholecalciferol is converted to 25-hydroxycholecalciferol in the kidneys
T b. 25 hydroxycholecalciferol converted to 1, 25 –dihydroxycholecalciferol in the kidneys
F c. 1, 25 –dihydroxycholecalciferol is stored in the liver for several months
F d. 1, 25 –dihydroxycholecalciferol is the least active form of vitamin D

291. the following are metabolic effects of corticosteroids :
T a. stabilization of lysosomal membranes
F b. increase capillary permeability
T c. suppression of T- cell activity
T d. reduction of fever

292. parathyroid hormone :
F a. is produced by the oxyphil cells of the parathyroid gland
F b. is released in response to increased plasma calcium concentration
T c. increases calcium absorption from the GI tract
F d. increases phosphate absorption from the renal tubules


293. glucagon :
T a. secretion is stimulated by amino acid alanine and arginine
T b. secretion is stimulated by exercise
T c. is a ketogenic hormone
F d. acts by increasing intracellular cGMP

294. the following stimulate insulin secretion :
T a. glucose
T b. cortisol
T c. amino acids
T d. gastrin

295. somatostatin :
T a. is a polypeptide hormone
T b. is produced by the delta cells of the islet of langerhans
T c. is produced by the hypothalamus
T d. inhibits growth hormone release

296. catecholamines :
F a. secretion is inhibited by hypoglycemia
T b. secretion is stimulated by hypothermia
F c. function through receptors located in the cytoplasm
F d. stimulate hepatic glycogen synthesis

297. during insulin synthesis and secretion :
T a. insulin is initially synthesized as a single chain per-pro-hormone
F b. pro-insulin has tow polypeptide chains linked by disulphide bonds
F c. in the secretory granules , four insulin molecules are attached to Zn to form a tetramer
F d. tow insulin molecules are secreted per molecule of C-peptide


298. The estrogen receptor:
F a. is localized to the nucleus only following ligand binding
F b. does not undergo nucleocytoplasmic shunting
F c. binds hear shock protein 90 on activation
T d. is regulated by phosphorylation

299. 5-alpha reductase:
F a. converts androgen to estrogens
T b. converts progesterone to dihydroprogesterone
T c. is required for androgen activity in hair follicles
F d. is not required for androgen activity on the penis

300. with respect to cortisol production by the feto-placental unit :
T a. maternal levels of CRH binding protein are lower in women with threatened pre-term labour who deliver within 24h
T b. maternal CRH levels increases with gestation age
T c. amniotic fluid levels of CRH binding protein fall before the onset of labour
F d. cord blood cortisol levels are lower after spontaneous labour compared to induced labour

301. human chorionic gondotrophins (HCG) :
F a. is metabolized and excreted mainly by proteolysis
T b. is excreted in urine mainly as the beta core fragment
F c. peak concentration in pregnancy is ~ 10,000 IU/L
T d. concentration is 10,000 – 20,000 IU/L at 18-20 weeks gestation

302. the progesterone receptor :
F a. when activated , is bound to heat shock protein -90
T b. when activated , forms adimer
F c. has an A sub –type which is the positive regulator of progesterone –sensitive genes
F d. is located in the cell membrane


303. Varicella zoster :
T a. causes shingles
T b. 85 % of pregnant women in the UK are immune to infection
T c. a history of previous chicken – pox indicates immunity
F d. can not be contracted from a patient with shingles


304. with respect to infection wilt cytomegalovirus :
T a. the virus is presnt in semen
F b. infection can not be sexually transmitted
F c. maternal immunity prevent congenital infection
F d. infection is effectively treated with acyclovir

305. influenza :
F a. is caused by an adenovirus
F b. infection is associated with a viraemia
T c. is sensitive to remantadine (amantadine)
T d. infection predispose to bacterial pneumonia

306. cytomegalovirus :
F a. causes a heterophile positive mononucleosis
F b. causes post-auricular and sub occipital
Lymphadenopathy
F c. primary infection occurs in 0.02 % of pregnant women
T d. is the most common congenital infection

307. Varicella zoster :
T a. is contagious until all vesicles have crusted over
T b severe infections causes adrenal insufficiency
F c. infection in pregnancy is associated with less sevre disease
F d. causes pneumonitis in 1% of infected pregnant women


308. pseudomonas aeruginosa :
F a. is sensitive to trimethoprim
F b. is sensitive to tetracycline
T c. is sensitive to gentamicin
T d. is sensitive to carbencillin

309. with respect to respect to staphylococci :
T a. S .aureus causes toxic shock syndrome
T b. S. aureus is a recognized cause of breast abscess
F c. methicillin resistant strains are sensitive to flucloxacilin
T d. methicillin resistant strains are sensitive to vancomycin

310. with respect to coliforms :
F a. shigella is part of normal faecal flora
T b. salmonella typhi may be found permanently in the biliary tract of carriers
F c. klebsiella characteristically sensitive to ampicillin
T d. coliforms are facultative anaerobes

311. bacteriodes :
T a. are sensitive to metronidazole
F b. are resistant to clindamycin
T c. are an important causes of pelvic abscesses
T d. are part of normal vaginal flora

312. legionella pneumophilia :
T a. is spread by contaminated shower mixers
F b. is sensitive to penicillin
F c. infection is treated with gentamicin
F d. is resistant to erythromycin



mandible
mandible
جني اصلي
جني اصلي

عدد المساهمات : 600
نقاط : 1800
السٌّمعَة : 9
تاريخ التسجيل : 2010-02-24

بطاقة الشخصية
gini: 4

https://mandible.yoo7.com

Back to top Go down

MCQS Part 1 MRCOG More than 300 stem Empty Re: MCQS Part 1 MRCOG More than 300 stem

Post by wafyazaz Fri Nov 12, 2010 10:42 pm

thank u
wafyazaz
wafyazaz
جني نشط
جني نشط

عدد المساهمات : 24
نقاط : 48
السٌّمعَة : 29
تاريخ التسجيل : 2010-11-12

Back to top Go down

MCQS Part 1 MRCOG More than 300 stem Empty Re: MCQS Part 1 MRCOG More than 300 stem

Post by drhmada Thu Feb 24, 2011 6:01 am

thank you from gaza
drhmada
drhmada
جني نشط
جني نشط

عدد المساهمات : 48
نقاط : 51
السٌّمعَة : 8
تاريخ التسجيل : 2010-11-18

Back to top Go down

MCQS Part 1 MRCOG More than 300 stem Empty Re: MCQS Part 1 MRCOG More than 300 stem

Post by drhmada Thu Feb 24, 2011 6:02 am

thank you from gaza
drhmada
drhmada
جني نشط
جني نشط

عدد المساهمات : 48
نقاط : 51
السٌّمعَة : 8
تاريخ التسجيل : 2010-11-18

Back to top Go down

MCQS Part 1 MRCOG More than 300 stem Empty Re: MCQS Part 1 MRCOG More than 300 stem

Post by drhmada Thu Feb 24, 2011 6:33 am

thank u gaza
drhmada
drhmada
جني نشط
جني نشط

عدد المساهمات : 48
نقاط : 51
السٌّمعَة : 8
تاريخ التسجيل : 2010-11-18

Back to top Go down

MCQS Part 1 MRCOG More than 300 stem Empty Re: MCQS Part 1 MRCOG More than 300 stem

Post by aml Mon Apr 04, 2011 5:26 pm

جزاك الله خيرا على هذه المجهود الرائع
ولكن الاسئلة الاولى اجاباتها غير موجودة فلو تكرمت وقمت بوضع الاجوبة نكون ممتنين لك.
ولو عند حضرتك اى نصائح ممكن افادتنا بها نكون شاكرين
aml
aml
جني
جني

عدد المساهمات : 15
نقاط : 18
السٌّمعَة : 8
تاريخ التسجيل : 2011-03-28

Back to top Go down

MCQS Part 1 MRCOG More than 300 stem Empty Re: MCQS Part 1 MRCOG More than 300 stem

Post by mujtaba Thu Aug 11, 2011 6:48 pm

شكرا
mujtaba
mujtaba
جني مبتدي
جني مبتدي

عدد المساهمات : 1
نقاط : 1
السٌّمعَة : 5
تاريخ التسجيل : 2011-08-11

Back to top Go down

MCQS Part 1 MRCOG More than 300 stem Empty Re: MCQS Part 1 MRCOG More than 300 stem

Post by drmamou Wed Oct 26, 2011 6:39 pm

مشكور على المجهود الرائع....و ياريت المزيد لو عندك مشان تعم الفائدة ..و ربنا يجعلها في ميزان حسناتك.
drmamou
drmamou
جني
جني

عدد المساهمات : 5
نقاط : 5
السٌّمعَة : 5
تاريخ التسجيل : 2011-10-26

Back to top Go down

MCQS Part 1 MRCOG More than 300 stem Empty NOTHING BETTER THAN SHARING IDEAS

Post by ahmed bassett Mon Nov 14, 2011 5:36 am

THANKS FOR THIS GOODQUESYION HOW TO FIND THE ANS MY SELF ...THEN I WILL ASK HELP THANKS A LOT
ahmed bassett
ahmed bassett
جني نشط
جني نشط

عدد المساهمات : 26
نقاط : 26
السٌّمعَة : 5
تاريخ التسجيل : 2011-11-13

Back to top Go down

MCQS Part 1 MRCOG More than 300 stem Empty جزاكم الله خيرا

Post by رهام Sun Jan 08, 2012 4:15 pm

mandible wrote:
[You must be registered and logged in to see this link.]

[PART 1 MRCOG FINAL ASSESSMENT
1.Which of the following will be seen in a villous of the placenta :
a. syncytiotrophoblast
b. cytotrophoblast
c. yolk
d. decidual cells
e. blood vessels

2. as pregnancy progresses, the amniotic fluid :
a. become more hypotonic
b. concentration of Na will falls
c. urea concentration increases
d. LFP concentration increase
e. PH become more acidic

3. These methods of placenta transfer are appropriate for the following substances :
a. oxygen _______________ active
b. fats __________________pinocytosis
c. iron __________________active
d. co2 ___________________ passive
e. antibodies _______________ pinocytosis

pharmacology

4. morphine :
a. the major pathway for it is detoxification is by oxidation to gentisic acid
b. increase biliary tract pressure
c. is eliminated mainly in the conjugated state via glomerular filtration
d. is secreted in the bile
e. produce excitement in male and female equally it does.

5. codeine
a. is not effective in large doses for severe pain
b. large doses cause excitement
c. large dose causes narcosis
d. is strong anittusive agent
e. aspirin and codeine are a useful combination against headache

6. the hazards of induction of anesthesia by injection of thiopentone are :
a. severe hypotension
b. cessation of respiration
c. necrosis following extra vascular injection
d. bronchospasm
e. stimulation of bronchial and salivary secretions

:
7. pethidine:
a. has analgesic properties .
b.is effective for biliary colic
c.will often relieve bronchial spasm
d.has quinidine like effect on myocardium
e. may cause hypotension

8. halothane :
a. is potent anesthetic and poor analgesic
b. cause uterine relaxation
c. cause liver failure in biliary tract disease
d. does not increase the catecholamine secretion
e. cause increased blood loss in therapeutic abortion


9. the gynecoid pelvis :
a. is the second commonest type of female pelvis
b. has wider anteroposterior than transverse diameter at the pelvic prim
c. has less prominent ischial spines than android pelvis
d. has ashallow ,wide sacrosciatic notch
e. has a parallel pelvic side wall .

10. the capsule of the hip joint :
a. contain fibrous bands which convey nutrient arteries supplying neck of femur
b. is strongest anteriorly and superiorly
c.is strongest posterior-inferior
d. cover the neck of the femur completely

11. the sacroiliac joint :
a. is asynovial joint
b. its main bond is the dorsal sacroiliac ligament
c . the interossious ligament is the strongest ligament of the joint
d. has anterioposterior rotary ligament
e. movements here are impaired by pregnancy

12. which of the following statements are correct in relation to the ischial spines :
a. they lie between the greater and lesser sciatic notches
b. they mark the begging of the forward curve of the birth canal
c. they are of particular prominence in platypelloid pelvis
d. when the head at its level , they consider that head is not yet engaged .
e.the internal pudendal nerve lies in close relationship to the spines

13. the following structures pass below the piriformis muscle through the greater sciatic notch :
a. sciatic nerve
b. the lateral femoral cutaneous nerve
c. the pudendal nerve
d. the nerve to the obturator externus
e. the inferior gluteal vessels

14. an ideal pelvis has :
a. a straight sacrum
b. an angle of inclination of 135
c. a subpubic angle of about 70
d. palpable sacral promontory .
e. parallel pelvic side walls


[You must be registered and logged in to see this link.]

Embryology

15. chromatin negative conditions include :
a. xx
b. xo
c. xxy
d. xxx
e. xy

16.differentiation of sexual organs :
a. is primarily directed by genetic factors
b. is influenced by supra renal cortex
c. is influenced by N((0)) of primordial germ cells
d. is influenced by over ripeness of ovum at fertilization
e. is not affected by chromosomal constitution

17. male and female analogues :
a. testis correspond to ovary
b. prostatic utricle corresponds to vagina
c. ventral penis corresponds to labia majora
d. distal male urethra corresponds to female vestibule
e. penis corresponds to crus clitoridis

18. the barr body :
a. arise from the Y chromosome
b. is found in turner syndrome
c. is found in testicular feminization S
d. occurs in 10 % of normal male
e. is found near the cell membrane on microscopic examination.

19. drugs which affect the fetus in late pregnancy :
a. oxytetracycline
b. long acting sulfa
c. streptomycin
d. ampicillin
e. erythromycin

20. cytotoxic drugs :
a. antimetabolites include methotrexate 5FU, 6 mercaptopurine
b. methotrexate block folic reductase
c. alkaylating agents include thiotepa and synthetic progestins
d. cycophosamide causes hemorrhagic cystitis
e. vincristine causes peripheral neuropathy

21. the drug of choice for typhoid fever :
a. sulphadiazine
b. penicillin
c. chloramphenicol
d. tetracycline
e. neomycin

22. the following karyotypes are appropriate :
a. turners syndrome 46 XO
b. testicular feminization 47 XY
c. klinfelters syndrome 47 XXY
d. super female 46 XXX
e.down syndrome 47 trisomy 21

23. ambiguous external genitalia at birth :
a. are commonly due to congenital adrenal hypoplasia
b. are associated with drug ingestion during pregnancy
c. occurs in testicular feminization syndrome
d. occurs in true hermaphroditism
e. are commonly associated with abnormal karyotype

24. the following effective against penicillinase producing staph :
a.cloxacillin
b. carboncillin
c. ampicillin
d. phenoxy methyl penicillin
e.flagyl

25. the following items are useful for prevention of side effect of chemotherapy :
a. full body irradiation
b.folinic acid
c. scalp cooling
d. corticosteroids
e. leucocytes transfusion

[You must be registered and logged in to see this link.]



26. the clotting mechanism :
a. the clotting factors may be assayed chemically
b. thrombin converts fibrinogen to fibrin monomer by splitting fibrinopeptides A + B
c. thrombin alters antihaemophilic factor VIII
d. thrombin is important in platelets haemostatic behavior
e. platelets accelerate the intirinsic clotting pathway by providing Ca
27.estrogen metabolism by the liver :
a. all the estrogens are inactivated by the liver
b. estrogen is 16 alpha- hydroxylated to 16 alpha hydroxytesterone
c. excretion in the bile is either in the conjugated or free form
d. in the liver cellular disease , impaired hydroxylation occur
e. urinary estrogen determination is a useful liver function test

[You must be registered and logged in to see this link.]

28. cholesterol :
a. is a constituent of most cell membranes
b. is a precursor of steroid only
c. synthesis take place mainly from acetate
d. synthesis occurs in microsomes and cell supply
e. bile salts are important un the regulation of its metabolism

29. amniotic fluid contains the following protein :
a. alpha 1 globulin
b. alpha 2 globulin
c. B globulin
d. Y globulin
e. all the above

30. normal haemopoiesis requires the following elements of vitamins :
a. iron
b. folic acid
c. cyanocobaamine
d. copper
e. manganese

31. is it true that :
a. insulin promote the uptakes of glucose by muscle cells
b. glucose 6- phosphatase occurs in muscle but not in liver
c. adrenalin promotes the breakdown of hepatic glycogen
d. lactate is the end product of the aerobic metabolism of glucose
e. insulin is secreted by the alpha cells of the islets of langerhans

32. which of the following are correct :
a. DNA is found in the nucleus and cytoplasm
b. RNA is found in the nucleus only
c. RNA is found in the cytoplasm only
d. RNA is found in the cytoplasm and nucleus
e. DNA is the chief constituent of ribosomes

33. which of the following are active process :
a. Ca ++ transport in the sarcoplasmic reticulum
b. Hcl secretion in the gastric epithelium
c. amino acid transport in the gut
d.glucose transport in the erythrocyte
e. C1/HCO exchange in erythrocytes


[You must be registered and logged in to see this link.]

34. statistics :
a. the median is a value such that 50 % of the observation fall above it and 50 % below it
b. the mode describes the most frequent value
c. the standard deviation measures scatter of distribution
d. the variance measures the greatest spread
e. sequential trials allow a trial to be discontinued at the earliest moment at which a judgment can be objectively formed

35. the standard deviation of a group of observation :
a. is the square of variance of this group
b. is a measure of the scatter observation around the mean
c. has a normal Gaussian distribution
d. may be used as the basis of the calculation of X 2
e. about 91 % of observation lie within 2 standard deviation of the mean




36.radiation
a. beta rays may be +ve or –ve electrone
b. gamma rays are similar to x-ray
c.a roentgen is equal to 48 ergs/gm
d. a rad measures emitted radiation
e. a rad is equal to 100 ergs for energy/gm

37. Radiation: the following isotopes have the half lives:
a. cesium 137 - 15 years
b. cobalt 60 - 5.2 years
c. radium 226 - 158 years
d. radium 222 - 1620 years
e. strontium 90 – 25 years

38. Respiratory changes in pregnancy:
a. with increasing pressure , the ribs flare out
b. the normal 168 subcostal angle increase to +_ 103 at turn
c. the diaphragm rise by +_ 4 cm
d.the transverse diameter of the chest increase by +_ 2 cm
e. respiratory is mainly diaphragmatic in pregnancy.
[You must be registered and logged in to see this link.]

39. the following respiratory functions increase in pregnancy:
a. vital capacity
b. tidal volume
c. residual volume
d. inspiratory capacity
E. minute ventilation

40. Cardiac action in pregnancy:
a. the mean COP is +_ 61/min
b. COP rises by +_1.5 L/M during 1st 10 weeks
c. COP rises by 1/3 during pregnancy
d. pulse rate rise by 1/5 during pregnancy
e. stroke volume falls during pregnancy

41. Effects of changed respiratory functions in pregnancy:
a. blood pH falls in pregnancy
b. plasma bicarbonate fall
c. plasma Na level fall
d. plasma osmolality fall
e. dyspnea is common even in absence of pathology

42. Erythrocytes & haemoglobin in pregnancy:
a. RBC fall from the average non – pregnant level of +_4.5 mill/ml to a minimum of +_3.5 mill/ml at about 30 weeks
b. the RBC count rises after 30 weeks
c. the hematocrit rises progressively to 38 weeks
d. erythrocytes tend to be more spherical in shape
e. Hb falls from 137 – 140 g/L to 110 – 120 g/L

43. Total body water:
a. may be as low as 30 % of body weight in newborn
b. is greater in male than female
c. is one half to 2/3 of body weight
d.is measured by dentrium oxide
e. is less in thin people

44. Lymphatics:
a. are characterized by their fast flow
b. are lined by endothelial cells
c. are permeable to protein
d. are scarce in cellular tissue
e. of intestine , transport and absorbed fat

45. Na reabsorption in proximal convoluted tubules:
a. is under hormonal control
b. is a passive process
c. is related to K secretion
d. is accompanied by passive absorption of CL
e. increased in pregnancy .

46. Which of the following dietary component are most likely to be inadequate in human pregnancy?
a. ascorbic acid
b. vitamin E
c. Ca++
b. Nacl
e. Iron

47. Oxygen dissociation curve shifts to left with:
a. raised temp
b. raised PCO2
c. reduced PH
d. anemia
e. stored blood

48. Hyperventilation:
a. in necessarily followed by period of apnea
b. decreased H2CO3 in the plasma
c. leads to tetany
d. increased Po2 of arterial blood
e. correct pH in metabolic acidosis

49. Veins:
a. of lower limbs have no valves
b. of neck are never collapse in erect position
c. are thin walled
d. Are innervated
e. of upper limb have no valves

50. Insulin cause:
a. increase utilization of glucose by the cells
b. increase glycogen store by the liver
c. decrease the amino acids and level in blood
d. decrease protein synthesis
e. decrease fatty acids levels in the blood

51. in respiratory acidosis:
a. the blood plasma reaction is acid
b. the kidney excrete an alkaline urine
c. the plasma bicarbonate is decreased
d. the primary defect is hypoventilation
e. cause can be ventilation/perfusion imbalance

52. Toxoplasmosis:
a. the dog is the definite host
b. the parasite exist in 2 forms cyst & oocyst
c. invasion is by the cyst
d. the oocyst is also important in human infection
e. cause cervical lymphadenopathy

53. The following diseases have an incubation period less than 10 days:
a. rabies
b. filariasis
c. dengue
d. yellow fever
e. hepatic amoebiasis

54. Anaerobic bacteria:
a. include gm +ve as well as gm –ve organism
b. produce hyaluronidase
c. bacteroid fragilis is most commonly seen
d. corticosteroid therapy lessen the chance of infection
e. the anaerobic gm –ve bacilli & anaerobic coccci are the major agent in aerobic infection
55. Hepatitis B antigen:
a. consist of Hbs Ag fraction
b. consist of Hbc Ag fraction
c. is only found in the serum sufferers (2-6) weeks after evidence of transaminase activity
d. occurs in 1/10000 of Caucasian
e. may spread in faeces

56. Structure of the simpler virus:
a. in the simpler viruses , the mature virus particle is the virion
b. the protein coat is the capsid
c. the centre is a single molecule of nuclei A
d. the nucleocapsid is the nucleic acid core plus the protein coat
e. capsids are composed of repeating units of one or a small number of polypeptides .

57. the agents of first choice for the organism mentioned are :
a. N. gonorrhea : cephalothin
b. T. palladium : tetracycline
c. C. wechiu : penicillin G
d. E. coli : gentamycin
e. S. aureus : methicillin, oxacillin or norfloxacin

58. consider cell necrosis , a nucleus which shrivels and become more dense is described as:
a. karyorhexis
b. karyolysis
b. pyknosis
d. vacular degeneration
e. condensation

59. Mismatched blood transfusion may lead to:
a. haemoglobinemia
b. haemoglobiuria
c. bilirubinuria
d. polycythaemia
e. renal failure

60. The characteristic reaction to be found in a tubercle usually includes:
a. pus cells
b. siderocyte
c. foreign body giant cells
d. epithelioid cells
e. lymphocytes

61. Multinucleated giant cells are seen in man in association with:
a. cat-scratch disease
b. measles
c. rheumatic fever
d. syphilis
e. TB

65. Necrosis is an important feature of the lesion in:
a. acute anterior poliomyelitis
b. labour pneumonia
c. rheumatic fever
d. tertiary syphilis
e. viral hepatitis

66. the following are true of a normal distribution :
a. 95 % of observations lie within mean +- 1 standard deviation (SD)
b. 2.5 % of observations lie below 2 standard deviation of the mean.
c. student s t-test could be used to compare the date with those of another population which is normally distributed
d. anon-parametric test may may be used to compare the data with those of another population .
e. 99 % of observational lie within 2.6 standard errors of the mean

67. The following statements are correct:
a. the standard error of the mean (SEM) provides an index of the spread of observations around the mean
b. the standard error of the mean is calculated as the square root of variance
c. the standard deviation is generally smaller than the standard error of the mean
d. the standard deviation is an index of the reliability of the mean
e. one advantage of the standard deviation is that it could be manipulated mathematically

68. Hyperpigmenation can occur in pregnancy in the following sites:
a. nipples and areolae
b. umbilicus
c. axillae
d. perineum
e. face
69. Alkalinization of the urine will enhance the excretion of:
a. phencyclidine
b. amiphetamine
c. phenobarbitone
d. aspirin
e. imipramine

70. Which of the following drugs may be administered orally with tetracycline?
a. magnesium trisilicate
b. aluminum hydroxide
c. salicylates
d. indomethacin
e. ferrous sulphate
71. Cell cycle specific antineuplastic agents act during following diseases:
a. DNA synthesis
b. transcription
c. function of the mitotic spindle
d. predna synthetic phase
e. G0 phase

72. Normal amniotic fluid contains:
a. glucose
b. fructose
c. lactose
d. lactic acid
e. citric acid

73. Concerning the physiology of amniotic fluid:
a. at term , it consist only of fetal urine
b. is removed mainly by fetal swallowing
c. is maximum at term
d. 400 – 500 ml are produced /24hrs
e. 5 ml/kg/hr is produced by a term fetus

74. which of the following separate maternal and fetal blood in human placenta :
a. trophoblast
b. maternal vascular endothelium
c. fetal vascular endothelium
d. maternal connective tissue
e. fetal connective tissue 75. Tinea capitis:
a. is very common in adult
b. can be caused by epidermophyton floccosum
c. can be easy treated topically
d. is common in children than adult
e. can occur in an epidemic

76. Hepatitis A is commonly transmitted by:
a. shared needles of drug users
b. faecal oral transfer
c. airborne respiratory droplets
d. blood transfusion
e. insect vectors
[You must be registered and logged in to see this link.]

77. Human papilloma viruses:
a. are typed by DNA hyperdization
b. are associated with condylomata acuminate
c. rarely cause infection in sexually active men
d. are associated with cervical cancer
e. should be subtyped for identification

78. digestive function in a healthy adult involves :
a. decreased production of saliva following parasympathetic activity
b. voluntary oesophageal contraction
c. release of gastrin from the antral portion of the stomach
d. delayed gastric emptying after a fatty meal
e. increased pepsin secretion with vagal stimulation

79. Monocytes in peripheral blood :
a. are less common than eosinophil granulocytes
b. are more common than basophil granulocytes
c. are phagocytic .
d. may migrate into tissues
e. are decreased in pregnancy

80. when blood vessel is injured :
a. factor XII is activated by collagen
b. factor VII is activated by thromboplastin
c. the intrinsic and extrinsic mechanisms share a common pathway following activation of factor VII
d. tissue factors are responsible for the speed of clot formation
e. the fibrin monomer first formed is soluble in plasma
81. comparing the male and female (gynaecoid ) bony pelvis :
a. the subpubic angle is greater in the female
b. in the female the diameter of the acetabulum is less than the distance between the acetabular rim and the symphysis pubis
c. the interspinous distance is greater in the female
d. the greater sciatic notch is larger in the male
e. the curvature of the sacrum is greater in the female

82. The peritoneum:
a. is lined by stratified squamous epithelium
b. has a visceral layer united firmly with underlying organs
c. has a parietal layer innervated by somatic spinal nerves
d. has venous drainage to the portal vein
e. can function as a dialyzing membrane

83. concerning the rectus sheath :
a. above the costal margin rectus abdominis lies on the costal cartilages
b. below the arcuate line the internal oblique splits to enclose rectus abdominis
c. it contains the musculo-phrenic artery
d. it is innervated by the ilio – inguinal nerve
e. pyramidalis is innervated by the subcostal nerve

84. The following drugs are effective systemically when given by mouth:
a. tetracycline hydrochloride
b. gentamicin sulphate
c. streptomycin
d. chloramphenicol
e. nifedipine

85. The administration of chloroquine phosphate may cause:
a. retinopathy
b. bleaching of the hair
c. changes in the cornea
d. deafness
e. photosensitivity

86. Calcification is associated with:
a. chronic renal failure
b. renal calculi
c. parathyroid adenoma
d. hyaline degeneration in uterine fibroids
e. secondary deposits in bone from prostatic carcinoma

87. collagen
a. is the most abundant protein in the body
b. contains hydroxyproline
c. depends in part on non-covalent bonds for its strength
d. is a component of basement membranes
e. is formed from procollagen

88. Wound healings is delayed by:
a. insulin
b. ultraviolet light
c. zinc deficiency
d. low temperature
e. glucocorticoids

89. Prostaglandins:
a. are synthesized from endoperoxides
b. have half live in the circulation of 6 hrs
c. are metabolized in the lungs
d. are synthesized in platelets
e. are synthesized by vascular endothelium

90. compare with cow milk, human milk contains more:
a. lactose
b. casein
c. sodium
d. water
e. ascorbic acid

91. The pelvic surface of the sacrum:
a. gives origin to the piriformis muscle
b. gives origin to the levator ani muscle
c. is broader in the male than in the female
d. transmit the dorsal rami of the sacral nerves
e. is in contact with the anal canal

92. Chromaffin cells:
a. are innervated by pre – ganglionic sympathetic nerve fibers
b. are present in the adrenal cortex
c. are derived from neuro- ectoderm
d. can decarboxylate amino acids
e. are present in celiac ganglia

93. in the fetal circulation:
a. the ductus venosus delivers blood directly into the superior vena cava
b. the umbilical artery returns blood from the placenta
c. the ductus aretriosus carries blood to the lungs
d. blood returning from the lungs is 90 % saturated with oxygen
e. blood from the inferior vena cava is largely directed through the foramen ovale

[You must be registered and logged in to see this link.]

94. The following structures take part in the formation of the anterior fontanelle in the fetal skull:
a. lambdoidal suture
b. occipital suture
c. sagittal suture
d. glabella
e. frontal suture
[You must be registered and logged in to see this link.]

95. The ductus venosus:
a. is part of the embryonic heart
b. is a shunt preventing blood from passing to the fetal lungs
c. gives rise to the legamentum teres
d. caries blood with a higher po2 than umbilical blood
e. is derived from the anterior cardinal vien

96. Successful lactation is:
a. maintained by estrogens
b. maintained by progesterone
c. initiated by a prolactin surge
d. maintained by human placental lactogen
e. inhibited by dopamine

97. in a woman of reproductive age, serum concentration of the following hormones exhibit a recognized pattern of diurnal variation:
a. progesterone
b. melatonin
c. cortisol
d. oestrone
e. follicle stimulating hormone

98. Serum concentration of the following increases during pregnancy:
a. sex hormone binding globulin
b. prolactin
c. total thyroxin
d. follicle stimulating hormone
e. 17 alpha – hydroxyprogesterone

99. Luteinzing hormone:
a. is required for normal corpus luteum survival
b. has a half life in the circulation of 30 hrs
c. is released in pulses
d. in the male stimulates testosterone production
e. plasma concentration are increased in post menopausal women

100. The secretion of growth hormone:
a. occurs in the hypothalamus
b. ceases when the adult stage is reached
c. is decreased during stress
d. is increased during fasting
e. is increased with exercise

101. Parathyroid hormone:
a. decrease the renal secretion of phosphate
b. increase calcium resorption in the bone
c. depresses pituitary activity
d. concentration in blood are raised when the calcium level falls
e. increases renal tubular resorption of calcium

102. Aldosterone:
a. reduce sodium resorption in the proximal convoluted tubules
b. reduces sodium absorption in the descending loop of Henle
c. increase sodium absorption in the distal convoluted tubules
d. increase potassium loss from the tubule
e. increase sodium absorption in the collecting tubule

103. The germination of tetanus spores in a wound is inhibited by :
a. tissue trauma
b. oxygen
c. injection of toxin
d. injection of toxoid
e. removal of devitalized tissue

104. cytomegalovirus :
a. is adenovirus
b. may be cultured readily in cell free media
c. is a cause of cerebral calcification
d. causes haemolytic anaemia in the neonate
e. may be transmitted in the saliva

105. hypokalaemia may be caused by :
a. bendrofluazide
b. digoxin
c. spironolactone
d. carbenoxolone
[You must be registered and logged in to see this link.]

e. amiloride

106. The following are required for haemostatic clot formation:
a. conversion of prothrombin to thrombin
b. platelet phospholipids
c. active conversion of plasminogen to plasmin
d. fibrin degradation product
e. antithrombin

107. Myometrial contractility:
a. is calcium dependant
b. is associated with phosphorylation of myosin light chain
c. is independent of cyclic adenosine monophosphate (cAMP)
d. is mediated by somatic nerves
e. depend on myometrial gap junctions

108. The following values fall within the normal range for the adult female bladder:
a. residual urine of 100 ml
b. voiding volume of 250 ml
c. bladder capacity of 900 ml
d. intravesical pressure rise to less than 10 cm H2O during early filling
e. maximum urine flow rate of 60 ml / second

109. in the testis:
a. maturation from spermatogonia to spermatozoa takes about 29 days
b. sertoli cells can mature into spermatids
c. leydig cells produce inhibin
d. luteinzing hormone inhibits the secretion of testosterone
e. large quantities of fructose are present in the seminal fluid

110. Function of the spleen in the healthy adult includes:
a. erythropoiesis
b. destruction of erythrocytes
c. formation of B lymphocytes
d. phagocytosis of bacteria
e. production erythropoietin


[You must be registered and logged in to see this link.]

111. metronidazole :
a. is effective against giardia lamblia
b. is effective when administered by rectum
c. should not be administered intravenously
d. is usually effective against Entamoeba histolytica
e. interfere with ethanol metabolism


[You must be registered and logged in to see this link.]

112. carcinoma in situ in epithelium ( intraepithelial neoplasia) is characterized by :
a. increased mitotic activity
b. loss of polarity
c. increased adhesiveness to underlying stroma
d. pyknosis
e. increased thickness of epithelium

113. the following provide conclusive evidence of pregnancy in uterine curetting :
a. deciduas compacta
b. arias-stella changes in endometrial glands
c. spiral arterioles
d. plasma cell infiltration
e. chorionic villi

114. Complications of myocardial infarction include:
a. fibrous pericarditis
b. aortic aneurysm
c. ventricular mural thrombi
d. coronary atherosclerosis
e. ventricular aneurysm

115. in the pathogenesis of thrombi :
a. prostacyclin induces platelet aggregation
b. platelets synthesized thromboxane A2
c. thromboxane A2 induces vasoconstriction
d. contact with subendothelial collagen causes platelet


116. the characteristic features of addisonian pernicious anaemia include :
a. leucocytosis
b. inheritance as an autosomal dominant trait
c. a raised corpuscular haemoglobin concentration
d. an increased incidence of gastric neoplasia
e. an increased incidence of primary hypothyroidism

117. immunodeficiency states may be associated with :
a. viral infection of T lymphocytes
b. B cell lymphomas
c. glucocorticoids administration
d. haemolytic disease of the new born
e. untreated Hodgkins lymphoma
[You must be registered and logged in to see this link.]


118. concerning immunoglobulins :
a. IgG contain tow heavy chains
b. IgM is produced before IgG in the immune response
c. IgE is the principle immunoglobulin secreted by mucous membranes
d. IgA is the principle immunoglobulin involved in allergic reactions
e. IgA is secreted in breast milk

119. antibodies :
a. are proteins
b. are formed in the fetus before 12 weeks of intrauterine life
c. have an average molecular weight of around 10,000 Daltons
d. of rhesus type are genetically transmitted
e. are produced by the ribosomes of plasma cells

120. the following disorders have an X linked pattern of inheritance :
a. glucose-6-phosphate dehydrogenase deficiency
b. kleinfelters syndrome
c. adrenogenital syndrome
d. haemophilia B
e. familial hypercholesterolemia

121. the following genetic disorders are inherited as autosomal recessives :
a. Duchene muscular dystrophy
b. Huntingdon’s chorea
c. tay-sachs disease
d. retinoblastoma
e. achondroplasia

122. concerning radiation physics :
a. an electron has a greater mass than a proton
b. a positron has the same charge as an electron
c. a proton has a positive charge
d. a neutron has almost the same mass as a proton
e. the hydrogen nucleus is a neutron

123. the conversion of glucose to lactic acid :
a. occur in a single enzymatic reaction
b. is the only pathway for the synthesis of ATP in the red blood cell
c. is a reversible process in skeletal muscle
d. is inhibited by high cellular concentration of ATP
e. occurs in skeletal muscle when the availability of oxygen is limited

124. normal human seminal fluid :
a. coagulates in vitro
b. conatins sucrose
c. has a pH of 5
d. may contain 15 % of morphologically abnormal spermatozoa
e. is predominantly produced within the testis

125. glucocorticoids :
a. promotes hepatic gluconeogenesis
b. suppress uptake of glucose by the muscle
c. promote protein breakdown
d. promote fat breakdown
e. increase glycolysis in adipose tissue

126. cholesterol :
a.Is essential dietary requirement
b. is present in the plasma membrane of all human cells
c. can not be synthesized by the liver
d. is transported in the circulation bound to albumin
e. is a precursor for the synthesis of steroid hormones .

127. fetal haemoglobin (HbF) :
a. is not formed before 20 weeks of intrauterine life
b. is more resistant than adult haemoglobin to denaturation by alkali
c. in the fetus constitutes 80-90 % of the haemoglobin at term
d. represent less than 5 % of total haemoglobin 8 weeks after birth
e. is increased in adult patients with beta thalassaemia

128. glucagon :
a. is a polypeptide hormone
b. is secreted by beta cells of the pancreatic islets
c. causes muscle glycogenolysis
d. has a half life of 5-10 minutes in the circulation
e. secretion is stimulated by cortisol

129. the female breast :
a. has a base which extend to the ninth rib
b. consist of up to 10 lobes
c. partially overlies the latissmus dorsi muscle
d. receives blood from the internal thoracic artery
e. is supplied in part by supraclavicular nerves

130. labia majora :
a. have a subcutaneous layer of non-striated muscle
b. have a fibro-fatty cord
c. contain the termination of the round ligament of the uterus
d. unite anteriorly forming the fourchette
e. have the openings of the greater vestibular glands on their medial surfaces .

131. the external dural (epidural) space :
a. contains the vertebral venous plexus
b. lie between pia and dura mater
c. contains no cerebrospinal fluid
d. ends at the level of the second lumbar vertebra
e. begins at the level of the first cervical vertebra

132. the following become occluded at or shortly after birth :
a. hepatic veins
b. ductus venosus
c. sinus venosus
d. ductus arteriosus
e. proximal ends of the hypogastric arteries

133. in the cell cycle :
a. DNA synthesis takes place during the M (mitosis) phase
b. RNA synthesis takes place predominantly during the S (synthesis ) phase
c. protein synthesis occurs during the G1 (pre-synthesis ) phase
d. microtubule assembly takes place during the G2 (post-synthesis ) phase
e. chromatids migrate to poles during metaphase

134. the peritoneum :
a. is lined by a stratified squamous epithelium
b. has a visceral layer united firmly with underlying organs
c. has a parietal layer innervated by somatic spinal nerves
d. has venous drainage to the portal vein
e. can function as a dialysing membrane

135. the following are clinical manifestation of infection of Coxsackie’s viruses :
a. aseptic meningitis
b. endometritis
c. epidemic myalgia
d. herpangina
e. pericarditis

136. prolactin secretion :
a. increases during sleep
b. increases during exercise
c. is increased by thyrotropin-realising hormone (TRH)
d. is decreased by dopamine
e. is increased by estrogen

137. in statistics a normal distribution :
a. has 50 % of its values on or above the mean
b. has 95 % of its values within tow standard deviations of the mean
c. has 10 % of its values below the 90th centile
d. has all its values below the third quartile
e. has 99 % of its values within three standard deviations of the mean

138. after the menopause :
a. calcium excretion decreases
b. there is a decreased secretion of follicle stimulating hormone
c. there is an increased secretion of progesterone
d. the vagina pH is increased
e. the endometrium becomes unresponsive to the action of estrogen

139. in the human female :
a. no new ova are formed after birth
b. miotic division is completed by puberty
c. prostaglandins are produced in the endometrium
d. average menstrual blood loss is about 150 ml
e. menstrual flow is maintained from the basal arteries


[You must be registered and logged in to see this link.]

140. the following viruses are commonly transmitted by coitus :
a. hepatitis B virus
b. human papilloma virus
c. herpes simplex virus
d. ebstein bar virus
e. cytomegalovirus

141. therapeutic substances predominantly metabolized by the liver include :
a. gentamicin
b. frusemide
c. morphine
d. phenothiazine
e. chlorpropamide

142. the following drugs are frequently used in the treatment of the named infections :
a. co-trimoxazole __ vaginal candidiasis
b. ampicillin – tuberculosis
c. tetracycline – trachoma
d. chloramphenicol – purulent conjunctivitis
e. trimethoprim – trichomoniasis

143. concerning haemoglobin :
a. haem contain ferric iron
b. the alpha globulin chain contains more than 100 amino acids
c. the affinity of haemoglobin for oxygen is modified by the red blood cell concentration of 2-3 , diphosphoglycerate
d. the oxygen affinity of deoxyhaemoglobin is greater than that of haemoglobin
e. a fall in pH shifts the haemoglobin oxygen dissociation curve to the left

144. Hereditary spherocytosis:
a. is inherited as a sex linked recessive characteristic
b. is associated with a deficiency of red cell pyrineratekinase
c. is associated with decreased fragility of red cells
d. may present in the first month of life
e. is associated with splenomegaly

145. stimulation of alpha adrenergic receptors :
a. does not occur with nor adrenaline
b. in the blood vessels of the skin leads to vasoconstriction
c. increases intestinal peristalsis
d. leads to contraction of erector pili muscles
e. leads to constriction of the pupil

146. epithelioid cells :
a. are transformed tissue macrophages
b. are found in Crohns disease
c. are found in granulation tissue
d. contains phagosomes (secondary lysosomes )
e. may undergo mitosis

147. in chronic infectious disease :
a. there is hyperplasia of the reticulo – endothelial system
b. specific IgG production is diminished
c. humorally mediated hypersensitivity reactions are common
d. anaemia is often of microcytic type
e. serum albumin levels are reduced

148. the following ions are predominantly extracelluar :
a. sodium
b. chloride
c. magnesium
d. phosphate
e. potassium

149. disease transmitted by insect vector :
a. trypansomiasis
b. yellow fever
c. achistosomiasis
d. leishmaniasis
e. filariasis

160. penicillin :
a. may be active against gram negative bacteria
b. interferes with normal cell wall synthesis
c. is inactivated by beta lactamase enzymes
d. is effective against resting bacterial cells
e. is contraindicated in pregnancy

161. the fetal testis :
a. are essential for the formation of the mesonephric (Wolffian) ducts
b. are essential of the formation of the paramesonephric (Mullerian)ducts
c. secretes testosterone
d. arise from the gonadal (genital) ridge
e. differentiate in response to a product of the Y chromosome

162. tetany may occur as a complication of :
a. osteoporosis
b. neonatal hypocalcaemia
c. respiratory alkalosis
d. peripheral neuropathy
e. untreated hyperparathyroidism

163. the tidal volume :
a. is about 700 ml at rest in pregnancy
b. is twice the anatomical dead space
c. together with the residual volume equals the vital capacity
d. is less than alveolar volume
e. is decreased by estrogens

164. the release of catecholamine from the adrenal medulla increases:
a. during sleep
b. when the nerves to the adrenal gland are stimulated
c. when the blood sugar rises
d. during excitement
e. in the presence of a phaeochromocytoma

165. the pineal gland :
a. is stimulated at the anterior end of the third ventricle
b. is innervated by the parasympathetic nervous system
c. produces melatonin
d. may be calcified in the adult
e. is most active during daylight

166. saliva :
a. contain an enzyme which is essential for the complete digestion of stomach
b. contains secretor substances
c. keeps the pH in the mouth between 5.0 and 6.0
d. has an antibacterial action
e. contains mucin

[You must be registered and logged in to see this link.]

167. the secretion of ant6idiuretic hormone is :
a. inhibited by ethyl alcohol
b. controlled by nervous stimuli from the hypothalamus
c. increased by a rising plasma osmolality
d. decreased by a falling plasma volume
e. inhibited by distension of the right atrium

168. mRNA :
a. is a double stranded polymer
b. is transcribed from DNA in the nucleus
c. is not present in reticulocytes
d. contains adenosine
e. is not present in oocytes

169. hypotension causes :
a. diminished urine production
b. inhibition of rennin secretion
c. stimulation of aldosterone secretion
d. stimulation of angiotensin II production
e. increased excretion of sodium

170. in the human the haploid number of chromosomes is found in the :
a. red blood cells
b. blastocyte
c. primary oocytes
d. 1st polar body
e. spermatozoon

171. the following statements relate to plasma proteins :
a. they create an oncotic pressure of 3.3 KPa (25 mm Hg ) across capillary walls
b. they form a major part of the plasma cationic pool
c. all are manufactured in the liver
d. albumin has a lower molecular weight than fibrinogen
e. fibrinogen is freely filtered at the glomerulus

172. in a healthy woman , rennin :
a. is secreted only by the kidney
b. plasma concentration is greater in the pregnant than in the non pregnant state
c. plasma concentration is increased by diuretic therapy
d. converts angiotensinogen into angiotensin II
e. activity is blocked by captopril

173. concerning human parturition :
a. the number of oxytocin receptors in the myometrium increases before the onset of labour
b. in the primigravida , cervical dilation usually precedes cervical effacement
c. the plasma oxytocin concentration increases at the onset of labour
d. oxytocin stimulates the synthesis of prostaglandins within the uterus
e. contraction of the maternal abdominal muscles is essential for spontaneous vaginal delivery

174. recognized effects of pregnancy include :
a. transient impairment of glucose tolerance
b. a raised glomerular filtration rate
c. a raised plasma concentration of free tyrosine
d. a reduced plasma concentration of alkaline phosphatase
e. an increased secretion of prolactin

175. during DNA replication :
a. only one strand of DNA is copied f
b. replication is semi conservative t
c. the DNA helix is unwound by the enzyme helicase t
d. DNA is synthesized by DNA synthetase f

176. the following are component of nucleotides :
a. purine or pyrimidine base t
b. pentose sugar t
c. phosphate group t
d. histones f


m
177. during DNA replication :
T a. okazaki fragments are later joined by DNA ligase
T b. DNA polymerase can detect base mis- match and correct any errors in DNA synthesis
F c. replication occurs during the G- phase of the cell cycle
T d. replication is bi-directional

178. with respect to transcription :
F a. the primary mRNA transcript is produced only from the exons on the DNA molecule
T b. introns have a sequence that begins with 5-GU
T c. introns have a sequence that ends with AG-3
T d. the removal of introns occurs in spliceosomes

179. transfer RNA :
F a. makes up to 80 % of total cellular RNA
T b. binds specific amino acids
F c. carries amino acids bind to its 5 end to polyribosomes
T d. has a clover-leaf secondary structure

180. the following are recognized post-translations modifications of proteins :
T a. formation of disulphide bonds
T b. excision off single sequences
T c. addition of oligosaccharides to asparagines residues
T d. proteolysis

181. with respect to protein synthesis from mRNA :
T a. each codon is specific for a particular amino acid
T b. one amino acid can be coded by more than one codon
F c. there are total of 48 possible codons
F d. during the synthesis of aminoacyl-tRNA from amino acids and tRNA , ATP is converted to ADP phosphate

182. during gluconeogenic phase of starvation :
F a. glycolysis is stimulated
T b.amino acids are released from skeletal muscle protein
F c. there is increased fatty acids synthesis
T d. phosphfructose is inactivated

183. during glycolysis :
F a. the initial reaction is the conversion of glucose to pyruvate
F b. glucose is converted to glucose-6- phosphate by glucose phosphatase
F c. the formation of glucose-6-phosphate from glucose requires the hydrolysis of ATP to AMP
T d. the formation of glucose-6-phosphate from glucose + ATP is an irreversible reaction

184. with respect to the regulation of glycogen metabolism :
T a. glycogen synthase is activated by dephosphorylation
T b. glucagon stimulates the phosphorylation of glycogen phosphorylase
T c. adrenaline stimulate the phosphorylation of glycogen synthase
F d. insulin result in the phosphorylation and inactivation of protein phosphatase 1

185. during glycogen breakdown :
F a. skeletal muscle glycogen can be converted to glucose and released into the circulation
T b. hepatic glycogen can be converted into glucose and released into the circulation
F c. the enzyme phosphorylase breaks alpha 1,4 glycosidic bonds less than 4 residues away from a branch point
F d. the enzyme phosphorylase requires the coenzyme biotin

186. with respect to the regulation of glycogen metabolism :
F a. insulin causes the dephosphorylation and activation of phosphorylase
T b. insulin causes the dephosphorylation and activation of glycogen synthase
T c. calcium ions stimulate glycogen breakdown in skeletal muscle
F d. skeletal muscle glycogen can be converted to glucose

187. the following are recognized classes of enzymes :
F a. digestive enzymes
F b. polymerases
T c. oxidoreductases
T d. transferases

188. enzymes :
T a. increase the rate at which a reaction reaches equilibrium
F b. shift the equilibrium point of a reaction towards the synthesis of product
F c. are inactivated at the end of the reaction
F d. increases the equilibrium constant of the reaction catalysed



189. with respect to inhibitors of enzyme action :
T a. irreversible inhibitors cause permanent inhibition of enzyme activity
T b. competitive inhibitors are usually structurally similar to the enzyme substrate
T c. the action of competitive inhibitor may be overcome by increasing substrate concentration
F d. competitive inhibitors are never converted to substrate

190. with respect to the kinetics of enzyme reaction :
T a. the Lineweaver-Burk plot has 1/initial velocity on the y-axis
T b. the sI unit of enzyme activity is the katal
F c. one katal is the quantity of enzyme in the presence of which one mole of substrate is converted per minute under optimal conditions
T d. the standard unit of enzyme activity U is the amount of enzyme catalysing the formation of one micromole of substrate per minute under optimal conditions

191. during the digestion of carbohydrates :
T a. salivary amylase hydrolyses alpha 1,4 glycosidic bonds in starch
F b. pepsin hydrolyses starch to disaccharides in the stomach
F c. pancreatic amylase has optimal enzyme activity at pH <3
F d. sucrase is secreted by the pancreas

192. vitamin B12 :
F a. can be synthesized de novo by humans
F b. is present in plant food
F c. is not stored by humans in significant amount
F d. is destroyed by cooking

193. thiamine :
F a. is not destroyed by cooking
F b. deficiency causes scurvy
T c. deficiency occurs in chronic alcoholics
T d. deficiency occurs in hyperemesis gravidarum

194. vitamin B12 deficiency occurs :
T a. in vegans
T b. following gastrectomy
F c. following biliary obstruction
T d. in celiac disease


195. vitamin D :
F a. metabolism by the kidney is principally regulated by calcitonin
F b. has its action through a membrane bound receptor
F c. deficiency causes renal failure
F d. deficiency causes osteoporosis

196. vitamin B12 :
T a. deficiency result in impaired DNA synthesis
T b. is a co-enzyme in the metabolism of metheyltetrahydrofolate
T c. requires intrinsic factor for it is absorption in the gut
F d. is transported bound to cyanocobalamine

197. necrosis :
F a. occurs when tissues are rapidly frozen
F b. is characterized by cytoplasmic basophilia
T c. may be associated by karyolyasis
T d. may be associated with karyorrhexis

198. dyspalsia :
T a. can be graded
T b. when severe represent carcinoma in situ
F c. does not occurs in the cervix
T d. can occur in the stomach

199. with respect to tissue calcification :
F a. metastatic calcification only occurs in malignant tissue
T b. metastatic calcification occurs in hyperparathyroidism
F c. metastatic calcification only occurs in damaged tissues
T d. metastatic calcification is associated with multiple myeloma

200. atrophy :
T a. is a decrease in the size of an organ during post natal life
T b. can occurs in the skeletal muscle
F c. is always physiological
F d. is associated with an increase in cell number

201. the following are characteristics of benign tumours :
F a. invasion into surrounding structures
T b. well defined capsules
F c. large size
F d. marked pleomorphism of the tumour cells


202. with respect to cervical intraepithelial neoplasia(CIN) :
F a. CIN I is characterised by the presence of dysplastic cells in the upper third of the epithelium
T b. CIN III is characterised by the presence of dysplastic cells throughout the epithelial layer
T c. CIN I may regress spontaneously
F d. CIN II is characterised by the presence of dysplastic cells in the upper tow thirds of the epitheliu


203. Cervical ectropion:
F a. is associated with dysplasia
F b. is caused by inversion of the distal endocervical canal
T c. is associated with use of the combined oral contraceptive pill
F d. is associated with human papilloma virus infection

204. Clear cell carcinoma of the vagina:
T a. is an adenocarcinoma
T b. is associated with vaginal adenosis
F c. typically present in post menopausal women
T d. is associated with in-utero exposure to diethylstilboestrol

205. Vulval intra-epithelial neoplasia:
T a. is premalignant
F b. is characterised by decreased nuclear cytoplasmic ratio in epithelial cells
T c. is associated with human papilloma virus infection
F d. is classified as VIN III if atypical cells occupy the lower third of the epithelium only

206. The following organs are typically involved in the direct invasion of cervical carcinoma:
T a. the ureter
T b. the bladder
F c. the internal iliac nodes
T d. the lower third of the vagina

207. Atypical endometrial hyperplasia:
F a. can readily be differentiated histologically from well differentiated endometrial carcinoma
F b. typically present around the menarche
F c. does not pro0gress into endometrial carcinoma
T d. typically present with peri – menopausal bleeding

208. uterine leiomyomas :
T a. typically reduce in size during menopause
F b. do not increase in size during pregnancy
F c. are pre malignant lesions
T d. are recognised cause of menorrhagia

209. adenomyosis :
T a. typically present with dysmenorrhoea
F b. is most common during menarche
T c. is the presence of endometrial tissue within the myometrium
F d. can be diagnosed at laparoscopy

210. chronic endometritis :
T a. is associated with infection with mycobacterium tuberculosis
T b. is characterised by lymphocytes and plasma cell infiltration
T c. can occur following miscarriage
T d. is associated with the intra – uterine contraceptive device

211. atypical endometrial hyperplasia :
F a. occurs as a progression of simple hyperplasia
T b. can occur in the absence of simple typerplasia
T c. is typically focal rather than diffuse
F d. is associated with stromal proliferation

212. the following are characteristic features of polycystic ovary syndrome :
F a. the presence of multiple 5 cm cysts in the ovary
T b. thickening of the ovarian capsule
T c. ovarian stromal hyperplasia
T d. leutenisation of the theca cells

213. carcino-embryonic antigen :
T a. is more likely to be elevated in women with mucinous compared to serous ovarian cancers
T b. is more likely to be activated in undifferentiated compared to well differentiated tumours
F c. is elevated in women with endometriosis
F d. is elevated in women with large uterine fibroids

214. CA – 125 levels are typically elevated in the following :
F a. ovarian terratomas
T b. ovarian hyperstimulation syndrome
T c. endometroid adenocarcinoma of the ovary
T d. pelvic inflammatory disease

215. with respect to hereditary ovarian cancer :
F a. 1% of ovarian cancers are hereditary
F b. the BRCA-1 mutation is inherited in an autosomal co-dominant fashion
F c. the prognosis of ovarian cancer is better in women with the BRCA-1 mutation than in those with sporadic cancers
T d. the BRCA-1 mutation is associated with an increased risk of colon cancer in males

216. ovarian thecomas :
T a. may be associated with pleural effusion
T b. are benign tumour
F c. secrete progesterone
F d. are cystic tumours

217. polymorphonuclear neutrophils :
F a. differentiate into monocytes
T b. are peroxidase positive
T c. are alkaline phosphatase positive
T d. contain collagen

218. tumour necrosis factor – alpha :
T a. is secreted by macrophages
F b. is secreted by neutrophils
T c. is secreted by monocytes
T d. induce cachexia

219. with respect to the complement system :
T a. the lectin pathway is a component of the innate immune system
T b. the lectin pathway is activated by the binding of mannose-binding lectin to the surface glycogen on microbes
T c. phagocytes express receptors for C3b
T d. C3b causes opsonization of bacteria

220. natural killer cells :
T a. do not express T- cell receptors
T b. are activated by interleukin-12
T c. are granular cells
T d. induce apoptosis in virally infected cells


221. polymorphonuclear neutrophils :
F a. stain with acid dyes
F b. stain with basic dyes
F c. divide by mitosis within the circulation
F d. are multi-nucleated

222. antibodies :
T a. serves as B-cell antigen receptors
T b. may be membrane-bound
T c. are present in breast milk
F d. do not cross the placenta

223. with respect to the structure of antibodies:
F a. the Fc fragment includes part of the light chain
T b. each antibody molecule has tow identical Fab regions
F c. each antibody molecule has tow Fc regions
T d. the hinge region is located between the Fab and Fc region


224. with respect to the peritoneal ligaments :
F a. the lateral umbilical ligament extend from the external iliac artery to the umbilicus
T b. the lateral umbilical ligament is obliterated umbilical artery
F c. the lienorenal ligament contains the renal artery
F d. the gastrosplenic ligament connects the lesser curvature of the stomach to the spleen

225. with respect to the duodenum :
F a. the head of the pancreas lies inferior to the third part of the duodenum
F b. the ligament of Treitz is attached to the third part of the duodenum
T c. the fourth part of the duodenum is related posteriorly to the aorta and left psoas muscle
T d. the third part of the duodenum is related posteriorly to the right ureter and right psoas muscle

225. with respect to the biliary tree :
T a. the bile canliculi drain into the interlobular ducts
F b. the interlobular ducts are located at the centre of the hepatic lobule
F c. the right hepatic duct drains bile from the caudate lobe of the liver
T d. the left hepatic duct drains bile from the quadrate lobe of the liver

226. the greater omentum :
T a. has its anterior layer attached to the greater curvature of the stomach
T b. has its posterior layer attached to the inferior border of the transverse colon
T c. has the lesser sac between the anterior and posterior layers
T d. contains the right gastro-epiploic artery

227. the cecum :
F a. is retroperitoneal
F b. has three bands of longitudinal muscles called appendices epiploicae
F c. has an outer continuous layer of longitudinal muscle
T d. does not have a mesentery


228. with respect to the surface anatomy of the abdomen :
T a. the sub –costal plane joins the lowest points of the costal margin on each side
F b. the sub –costal plane is at the level of the body of the 5th lumbar vertebra
F c. the inter-tubercular plane joins the ischial tuberosities on each side
T d. the inter-tubercular plane is at the level of the body of the 5th lumbar vertebra

229. with respect to the muscles of the posterior abdominal wall :
F a. psoas major inserts onto the greater trochanter
T b. iliacus inserts onto the lesser trochanter
T c. iliacus is supplied by the femoral nerve
T d. iliacus flexes the thigh on the trunk

230. with respect to the scrotum and its contents :
T a. dartos muscle is smooth muscle
F b. the external spermatic fascia lies superficial to dartos muscle
T c. cremasteric fascia lies deep to the external spermatic fascia
F d. the tunica vaginalis cover the anterior , posterior and lateral aspects of the testis

231. with respect to the scrotum and its contents :
F a. the lymphatic drainage of the scrotum is to the deep inguinal nodes
F b. lymphatic drainage of the testis is to the Para-aortic nodes at the level of L5
F c. the temperature in the scrotum is maintained at 3C above body temperature
F d. the pampiniform plexus is formed by the testicular artery

232. the following structures are present within the inguinal canal in the male :
T a. testicular artery
T b. vas deferens
F c. inguinal nerve
T d. genital branch of the genitor-femoral nerve

[You must be registered and logged in to see this link.]

233. the portal vein :
F a. is formed by the union of the superior and inferior mesenteric veins
F b. lies anterior to the first part of the duodenum
T c. lies within the lesser omentum
F d. lies posterior to the opening into the lesser sac

234. with respect to the blood supply to the gastrointestinal tract :
F a. the inferior pancreatico-dudenal artery is a branch of the celiac artery
F b. the middle colic artery is a branch of the inferior mesenteric artery
T c. the ileocolic artery is a branch of the superior mesenteric artery
T d. the inferior mesenteric artery is the artery to the hind gut

235.with respect to the lumbar plexus :
T a. the genitor –femoral nerve derived from L1 & L2
T b. the genitor-femoral nerve emerges from the lumbar plexus on the anterior surface of psoas major
T c. the genital branch of the genitor-femoral nerve enters the spermatic cord
F d. the femoral branch of the genitor –femoral nerve supplies the cremasteric muscle

236.the inferior vena cava within the abdomen :
T a. lies to the right of the aorta
F b. pierces the central tendon of the diaphragm at the level of T12
T c. lies medial to the right ureter
T d. is related anteriorly to the opening into the lesser sac

237.the celiac artery :
F a. is the artery of the mid gut
F b. lies anterior to the lesser sac
T c. is a branch of the abdominal aorta at the level of the 12th thoracic vertebra
F d. has three branches – the right gastric artery, the splenic artery and the hepatic artery

238. with respect to the sacral plexus :
F a. the sciatic nerve is formed from S 1,2&3
T b. the sciatic nerve leave the pelvis through the greater sciatic foramen
T c. the superior gluteal nerve is the largest nerve in the body
F d. the superior gluteal nerve leaves the pelvis through the lesser sciatic foramen

239.the levator ani muscle :
T a. originates from the back of the pubic bone anteriorly
T b. originates from the obturator fascia laterally
T c. originates from the ischial spine
F d. the anterior fibres are inserted onto the symphysis pubis

240. with respect to the hypogastric plexuses :
F a. the superior hypogastric plexus lies in the retro-peritoneal tissue between the tow internal iliac arteries
T b. the superior hypogastric plexus lies anteriorly
T c. . the superior hypogastric plexus contains both sympathetic and Para sympathetic fibres
F d. . the superior hypogastric plexus receives branches from the pelvic aplanchnic nerves

241. with respect to the sacrum :
T a. the sacral canal is formed by the vertebral foramina of the sacral vertebrae
F b. the sacral canal contains the spinal cord
T c. the sacral canal contains the filum terminale as far down as the level of S5
T d. the sacral canal contains the cauda equine as far down as the level of S2

242. the pelvic splanchnic nerves :
T a. join the inferior hypogastric plexus directly
F b. join the superior hypogastric plexus directly
T c. provide para sympathetic supply to the hind gut
F d. provide sympathetic supply to the bladder

243. the sigmoid colon :
T a. receives its nerve supply from the inferior hypogastric plexuses
F b. has an outer continuous layer of longitudinal smooth muscle
T c. lymphatic drainage is to the inferior mesenteric nodes
F d. is supplied by the sigmoid branches of the internal iliac artery

244. the cervix :
F a. is supplied by the cervical artery , a branch of the internal iliac artery
F b. receives nerve supply from the pudendal nerve
T c. undergoes cyclical changes during the menstrual cycle
F d. sheds its lining during menstruation
245. the following normal structures can be palpated through the vagina :
F a. the ureters
T b. the ovaries
F c. the fallopian tubes
T d. the uterus

246. the uterine artery :
F a. is a branch of the posterior division of the internal iliac artery
F b. is crossed by the ureter
F c. runs medially in the infudibulo- pelvic ligament
F d. reaches the pelvis at the level of the external os

247. with respect to the rectum :
F a. The middle third is covered by peritoneum on its anterior and lateral surfaces
F b. the lower third is covered by peritoneum on its anterior surface only
T c. the lumen has tow transverse folds on the left and one on the right rectal wall
F d. lymhatics from the lower third of the rectum drain into the external iliac nodes

248. the superficial perineal pouch in the female :
T a. contains the bulb of the vestibule
F b. lies between the tow layers of the urogenital diaphragm
T c. contains the perineal branch of the pudendal nerve
T d. contains branches of the internal pudendal artery

249. the bulbospongiosus muscle :
F a. surround the orifice of the urethra
T b. surround the orifice of the vagina
T c. covers the bulb of the vestibule
F d. is attached to the crus of the clitoris

250. the superficial transverse perineal muscle :
F a. lies in the anterior part of the superficial perineal pouch
F b. originates from the ischial tuberosity
T c. is inserted onto the perineal body
T d. is supplied by the perineal branch of the pudendal nerve



251. the pudendal nerve :
T a. supply the mucus membrane of the lower half of the anal canal
T b. supplies the perineal skin
T c. supplies the clitoris
F d. gives off the superior rectal nerve

252. with respect to the external genitalia in the female :
T a. the vestibule is triangular in shape
F b. the vestibule is bounded anteriorly by the perineal body
F c. the opening of the urethra lies anterior to the clitoris
F d. the vestibule is bounded laterally by the labia majora


253. the rib :
T a. has a head which articulate with a vertebral body
T b. has a tubercle which articulates with the transverse processes of the numerically corresponding vertebra
F c. has an angle between the head and the tubercle
F d. has a neck between the tubercle and the angle

254. the diaphragm :
F a. has a central muscular part and a peripheral tendinous part
F b. has a sternal origin from the p[posterior surface of the body of the sternum
F c. has a costal origin from the deep surfaces of the lower 4 ribs and their costal cartilages
F d. has a right crus originating from the medial arcuate ligaments

255. the mammary glands :
T a. receives arterial supply from the internal thoracic artery
T b. receives arterial supply from the axillary artery
T c. receives arterial supply from the inter costal arteries
F d. divided into 5-10 lobes radiating from the nipple

256. the diaphragm :
F a. is covered by visceral pleura on its superior surface
T b. receives motor nerves supply from the phrenic nerve
T c. receives sensory nerves supply from the phrenic nerve
T d. receives sensory supply from the lower 5 inter costal nerves



257. The first rib :
T a. articulate with the first thoracic vertebra
T b. articulates with the body of the 7th cervical vertebra
T c. is crossed by the subcalvian vein anterior to the scalene tubercle
F d. is crossed by the subcalvian artery posterior to the scalene tubercle

258. the right ventricle :
F a. has a pulmonary out flow tract (pulmonary valve ) with tow semilunar caps
F b. contracts during ventricular diastole
F c. communicates directly with the inferior vena cava
F d. pumps oxygenated blood into the pulmonary artery

259. with respect to the surface markings of the heart valves :
T a. the tricuspid valve lies behind the right half of the sternum opposite the 4th inter costal space
T b. the mitral valve lies behind the left half of the sternum opposite the 4th inter costal cartilage
T c. the pulmonary valve lies behind the medial end of the third left costal cartilage
T d. the aortic valve lies behind the left half of the sternum opposite the third intercostal space

260. with respect to the conducting system of the heart :
T a. the Sino atrial nodes is made of specialized cardiac muscle
F b. the Sino atrial nodes is located within the wall of the left atrium
T c. the atrio ventricular nodes is located in the lower part of the atrial septum
F d. the Sino atrial nodes is connected to the atrio ventricular node by the atrio ventricular bundle

261. the left ventricle :
F a. pumps deoxygenated blood to the lungs
F b. communicates with the right ventricle
F c. communicates with the aorta via the aortic valve which has tow semilunar cusps
T d. communicates with the left atrium via mitral valve which has tow cusps


262. the left ventricle :
T a. has trabeculae carneae
F b. has moderator band
F c. has three papillary muscles
T d. has a thicker wall than that of the right ventricle


263. the following are derivatives of neural crest :
F a. sensory ganglia of all the cranial nerves
T b. melanocytes
T c. the cartilage of the pharyngeal arches
T d. schwann cells

264. the primitive streak :
F a. appears in the surface of the hypoblast on day 15-16
F b. has the primitive node located at its caudal end
F c. formation leads to gastrulation and the formation of the extra-embryonic mesoderm
T d. formation leads to gastrulation and the formation of the intra- embryonic endoderm

265. with respect to the process of oogenesis :
F a. the zona pellucida develops after ovulation
F b. the first meiotic division is completed before puberty
F c. the second meiotic divisions is completed before ovulation
T d. the second meiotic division is completed only if the oocyte is fertilized

266. fertilization :
T a. occurs in the ampulla of the fallopian tube
F b. results in completion of the first meiotic division in the oocyte
F c. results in the cortical granules of the oocyte being released into the cytoplasm
T d. occurs before the completion of the second meiotic division in the oocyte

267. the following are derivatives of the neural crest :
T a. dorsal root ganglia
T b. pre aortic sympathetic ganglia
T c. the sympathetic chain
T d. parasympathetic ganglia














[right][justify][left]
رهام
رهام
جني مبتدي
جني مبتدي

عدد المساهمات : 2
نقاط : 4
السٌّمعَة : 5
تاريخ التسجيل : 2011-11-19

Back to top Go down

MCQS Part 1 MRCOG More than 300 stem Empty Re: MCQS Part 1 MRCOG More than 300 stem

Post by salymashaly Wed Feb 01, 2012 2:01 am

انا د.سالى مصريه بشكركم جدا على المجهود الرائع ولو سمحتوا عايزها اجابه الاسئله علشان توفير للوقت مشكورين جدا
salymashaly
salymashaly
جني
جني

عدد المساهمات : 5
نقاط : 5
السٌّمعَة : 5
تاريخ التسجيل : 2012-01-23

Back to top Go down

MCQS Part 1 MRCOG More than 300 stem Empty Re: MCQS Part 1 MRCOG More than 300 stem

Post by salymashaly Wed Feb 01, 2012 2:08 am

بالنسبه للالف سؤال مش فاهمه شروط تنزيلها علشان انا محتاجه لهم ضرورى جدا و هيفرق معايا جامد جدا عدم الحصول عليهم ردوا عليه مشكورين
salymashaly
salymashaly
جني
جني

عدد المساهمات : 5
نقاط : 5
السٌّمعَة : 5
تاريخ التسجيل : 2012-01-23

Back to top Go down

MCQS Part 1 MRCOG More than 300 stem Empty Re: MCQS Part 1 MRCOG More than 300 stem

Post by salymashaly Tue Feb 14, 2012 1:30 am

لو سمحتوا ماجورين ومشكورين انا محتاجه ضرورى جدا الاجابه على الاسئله كلها علشان مفيش وقت ابحث عنها و انا داخله مارس اللى جاى ارجوكوا ساعدونى
salymashaly
salymashaly
جني
جني

عدد المساهمات : 5
نقاط : 5
السٌّمعَة : 5
تاريخ التسجيل : 2012-01-23

Back to top Go down

MCQS Part 1 MRCOG More than 300 stem Empty Re: MCQS Part 1 MRCOG More than 300 stem

Post by د/ يوسف النو Mon Apr 09, 2012 1:34 am

شكرا عزيزي نجم علي المجهود الجبار وربنا يجزيك خير
د/ يوسف النو
د/ يوسف النو
جني مبتدي
جني مبتدي

عدد المساهمات : 1
نقاط : 1
السٌّمعَة : 5
تاريخ التسجيل : 2012-04-08

Back to top Go down

MCQS Part 1 MRCOG More than 300 stem Empty Re: MCQS Part 1 MRCOG More than 300 stem

Post by mandible Mon Apr 09, 2012 2:58 am

د/ يوسف النو wrote:شكرا عزيزي نجم علي المجهود الجبار وربنا يجزيك خير
شكرًا د. يوسف ونرجو مشاركتكم
mandible
mandible
جني اصلي
جني اصلي

عدد المساهمات : 600
نقاط : 1800
السٌّمعَة : 9
تاريخ التسجيل : 2010-02-24

بطاقة الشخصية
gini: 4

https://mandible.yoo7.com

Back to top Go down

MCQS Part 1 MRCOG More than 300 stem Empty Re: MCQS Part 1 MRCOG More than 300 stem

Post by marwa88 Mon May 07, 2012 6:10 am

ازيكم يا شباب انا جديده في المنتدى وعايزه استفسر عن بعض الحاجات
اولا انا اتخرجت شهر1 وطبعا لسه مااتوزعت للامتياز وعايزه اعمل part1 obse وعايزه اعرف اقرب امتحان متين والكتب المفروض اقرا منها شنو
وياريييييت تساعدوني وتردو علي اكون شاكره ليكم جداجداجدا
marwa88
marwa88
جني مبتدي
جني مبتدي

عدد المساهمات : 3
نقاط : 3
السٌّمعَة : 5
تاريخ التسجيل : 2012-05-06
العمر : 36
الموقع : makkah

Back to top Go down

MCQS Part 1 MRCOG More than 300 stem Empty Re: MCQS Part 1 MRCOG More than 300 stem

Post by ahlam qabaja Sat Mar 15, 2014 2:42 am

مرحبا يعطيكم العافيه بس ياريت تحلو الاسئله دي
1.Which of the following will be seen in a villous of the placenta :
a. syncytiotrophoblast
b. cytotrophoblast
c. yolk
d. decidual cells
e. blood vessels

2. as pregnancy progresses, the amniotic fluid :
a. become more hypotonic
b. concentration of Na will falls
c. urea concentration increases
d. LFP concentration increase
e. PH become more acidic

3. These methods of placenta transfer are appropriate for the following substances :
a. oxygen _______________ active
b. fats __________________pinocytosis
c. iron __________________active
d. co2 ___________________ passive
e. antibodies _______________ pinocytosis

pharmacology

4. morphine :
a. the major pathway for it is detoxification is by oxidation to gentisic acid
b. increase biliary tract pressure
c. is eliminated mainly in the conjugated state via glomerular filtration
d. is secreted in the bile
e. produce excitement in male and female equally it does.

5. codeine
a. is not effective in large doses for severe pain
b. large doses cause excitement
c. large dose causes narcosis
d. is strong anittusive agent
e. aspirin and codeine are a useful combination against headache

6. the hazards of induction of anesthesia by injection of thiopentone are :
a. severe hypotension
b. cessation of respiration
c. necrosis following extra vascular injection
d. bronchospasm
e. stimulation of bronchial and salivary secretions

:
7. pethidine:
a. has analgesic properties .
b.is effective for biliary colic
c.will often relieve bronchial spasm
d.has quinidine like effect on myocardium
e. may cause hypotension

8. halothane :
a. is potent anesthetic and poor analgesic
b. cause uterine relaxation
c. cause liver failure in biliary tract disease
d. does not increase the catecholamine secretion
e. cause increased blood loss in therapeutic abortion


9. the gynecoid pelvis :
a. is the second commonest type of female pelvis
b. has wider anteroposterior than transverse diameter at the pelvic prim
c. has less prominent ischial spines than android pelvis
d. has ashallow ,wide sacrosciatic notch
e. has a parallel pelvic side wall .

10. the capsule of the hip joint :
a. contain fibrous bands which convey nutrient arteries supplying neck of femur
b. is strongest anteriorly and superiorly
c.is strongest posterior-inferior
d. cover the neck of the femur completely

11. the sacroiliac joint :
a. is asynovial joint
b. its main bond is the dorsal sacroiliac ligament
c . the interossious ligament is the strongest ligament of the joint
d. has anterioposterior rotary ligament
e. movements here are impaired by pregnancy

12. which of the following statements are correct in relation to the ischial spines :
a. they lie between the greater and lesser sciatic notches
b. they mark the begging of the forward curve of the birth canal
c. they are of particular prominence in platypelloid pelvis
d. when the head at its level , they consider that head is not yet engaged .
e.the internal pudendal nerve lies in close relationship to the spines

13. the following structures pass below the piriformis muscle through the greater sciatic notch :
a. sciatic nerve
b. the lateral femoral cutaneous nerve
c. the pudendal nerve
d. the nerve to the obturator externus
e. the inferior gluteal vessels

14. an ideal pelvis has :
a. a straight sacrum
b. an angle of inclination of 135
c. a subpubic angle of about 70
d. palpable sacral promontory .
e. parallel pelvic side walls


[You must be registered and logged in to see this link.]

Embryology

15. chromatin negative conditions include :
a. xx
b. xo
c. xxy
d. xxx
e. xy

16.differentiation of sexual organs :
a. is primarily directed by genetic factors
b. is influenced by supra renal cortex
c. is influenced by N((0)) of primordial germ cells
d. is influenced by over ripeness of ovum at fertilization
e. is not affected by chromosomal constitution

17. male and female analogues :
a. testis correspond to ovary
b. prostatic utricle corresponds to vagina
c. ventral penis corresponds to labia majora
d. distal male urethra corresponds to female vestibule
e. penis corresponds to crus clitoridis

18. the barr body :
a. arise from the Y chromosome
b. is found in turner syndrome
c. is found in testicular feminization S
d. occurs in 10 % of normal male
e. is found near the cell membrane on microscopic examination.

19. drugs which affect the fetus in late pregnancy :
a. oxytetracycline
b. long acting sulfa
c. streptomycin
d. ampicillin
e. erythromycin

20. cytotoxic drugs :
a. antimetabolites include methotrexate 5FU, 6 mercaptopurine
b. methotrexate block folic reductase
c. alkaylating agents include thiotepa and synthetic progestins
d. cycophosamide causes hemorrhagic cystitis
e. vincristine causes peripheral neuropathy

21. the drug of choice for typhoid fever :
a. sulphadiazine
b. penicillin
c. chloramphenicol
d. tetracycline
e. neomycin

22. the following karyotypes are appropriate :
a. turners syndrome 46 XO
b. testicular feminization 47 XY
c. klinfelters syndrome 47 XXY
d. super female 46 XXX
e.down syndrome 47 trisomy 21

23. ambiguous external genitalia at birth :
a. are commonly due to congenital adrenal hypoplasia
b. are associated with drug ingestion during pregnancy
c. occurs in testicular feminization syndrome
d. occurs in true hermaphroditism
e. are commonly associated with abnormal karyotype

24. the following effective against penicillinase producing staph :
a.cloxacillin
b. carboncillin
c. ampicillin
d. phenoxy methyl penicillin
e.flagyl

25. the following items are useful for prevention of side effect of chemotherapy :
a. full body irradiation
b.folinic acid
c. scalp cooling
d. corticosteroids
e. leucocytes transfusion

[You must be registered and logged in to see this link.]



26. the clotting mechanism :
a. the clotting factors may be assayed chemically
b. thrombin converts fibrinogen to fibrin monomer by splitting fibrinopeptides A + B
c. thrombin alters antihaemophilic factor VIII
d. thrombin is important in platelets haemostatic behavior
e. platelets accelerate the intirinsic clotting pathway by providing Ca
27.estrogen metabolism by the liver :
a. all the estrogens are inactivated by the liver
b. estrogen is 16 alpha- hydroxylated to 16 alpha hydroxytesterone
c. excretion in the bile is either in the conjugated or free form
d. in the liver cellular disease , impaired hydroxylation occur
e. urinary estrogen determination is a useful liver function test

[You must be registered and logged in to see this link.]

28. cholesterol :
a. is a constituent of most cell membranes
b. is a precursor of steroid only
c. synthesis take place mainly from acetate
d. synthesis occurs in microsomes and cell supply
e. bile salts are important un the regulation of its metabolism

29. amniotic fluid contains the following protein :
a. alpha 1 globulin
b. alpha 2 globulin
c. B globulin
d. Y globulin
e. all the above

30. normal haemopoiesis requires the following elements of vitamins :
a. iron
b. folic acid
c. cyanocobaamine
d. copper
e. manganese

31. is it true that :
a. insulin promote the uptakes of glucose by muscle cells
b. glucose 6- phosphatase occurs in muscle but not in liver
c. adrenalin promotes the breakdown of hepatic glycogen
d. lactate is the end product of the aerobic metabolism of glucose
e. insulin is secreted by the alpha cells of the islets of langerhans

32. which of the following are correct :
a. DNA is found in the nucleus and cytoplasm
b. RNA is found in the nucleus only
c. RNA is found in the cytoplasm only
d. RNA is found in the cytoplasm and nucleus
e. DNA is the chief constituent of ribosomes

33. which of the following are active process :
a. Ca ++ transport in the sarcoplasmic reticulum
b. Hcl secretion in the gastric epithelium
c. amino acid transport in the gut
d.glucose transport in the erythrocyte
e. C1/HCO exchange in erythrocytes


[You must be registered and logged in to see this link.]

34. statistics :
a. the median is a value such that 50 % of the observation fall above it and 50 % below it
b. the mode describes the most frequent value
c. the standard deviation measures scatter of distribution
d. the variance measures the greatest spread
e. sequential trials allow a trial to be discontinued at the earliest moment at which a judgment can be objectively formed

35. the standard deviation of a group of observation :
a. is the square of variance of this group
b. is a measure of the scatter observation around the mean
c. has a normal Gaussian distribution
d. may be used as the basis of the calculation of X 2
e. about 91 % of observation lie within 2 standard deviation of the mean




36.radiation
a. beta rays may be +ve or –ve electrone
b. gamma rays are similar to x-ray
c.a roentgen is equal to 48 ergs/gm
d. a rad measures emitted radiation
e. a rad is equal to 100 ergs for energy/gm

37. Radiation: the following isotopes have the half lives:
a. cesium 137 - 15 years
b. cobalt 60 - 5.2 years
c. radium 226 - 158 years
d. radium 222 - 1620 years
e. strontium 90 – 25 years

38. Respiratory changes in pregnancy:
a. with increasing pressure , the ribs flare out
b. the normal 168 subcostal angle increase to +_ 103 at turn
c. the diaphragm rise by +_ 4 cm
d.the transverse diameter of the chest increase by +_ 2 cm
e. respiratory is mainly diaphragmatic in pregnancy.
[You must be registered and logged in to see this link.]

39. the following respiratory functions increase in pregnancy:
a. vital capacity
b. tidal volume
c. residual volume
d. inspiratory capacity
E. minute ventilation

40. Cardiac action in pregnancy:
a. the mean COP is +_ 61/min
b. COP rises by +_1.5 L/M during 1st 10 weeks
c. COP rises by 1/3 during pregnancy
d. pulse rate rise by 1/5 during pregnancy
e. stroke volume falls during pregnancy

41. Effects of changed respiratory functions in pregnancy:
a. blood pH falls in pregnancy
b. plasma bicarbonate fall
c. plasma Na level fall
d. plasma osmolality fall
e. dyspnea is common even in absence of pathology

42. Erythrocytes & haemoglobin in pregnancy:
a. RBC fall from the average non – pregnant level of +_4.5 mill/ml to a minimum of +_3.5 mill/ml at about 30 weeks
b. the RBC count rises after 30 weeks
c. the hematocrit rises progressively to 38 weeks
d. erythrocytes tend to be more spherical in shape
e. Hb falls from 137 – 140 g/L to 110 – 120 g/L

43. Total body water:
a. may be as low as 30 % of body weight in newborn
b. is greater in male than female
c. is one half to 2/3 of body weight
d.is measured by dentrium oxide
e. is less in thin people

44. Lymphatics:
a. are characterized by their fast flow
b. are lined by endothelial cells
c. are permeable to protein
d. are scarce in cellular tissue
e. of intestine , transport and absorbed fat

45. Na reabsorption in proximal convoluted tubules:
a. is under hormonal control
b. is a passive process
c. is related to K secretion
d. is accompanied by passive absorption of CL
e. increased in pregnancy .

46. Which of the following dietary component are most likely to be inadequate in human pregnancy?
a. ascorbic acid
b. vitamin E
c. Ca++
b. Nacl
e. Iron

47. Oxygen dissociation curve shifts to left with:
a. raised temp
b. raised PCO2
c. reduced PH
d. anemia
e. stored blood

48. Hyperventilation:
a. in necessarily followed by period of apnea
b. decreased H2CO3 in the plasma
c. leads to tetany
d. increased Po2 of arterial blood
e. correct pH in metabolic acidosis

49. Veins:
a. of lower limbs have no valves
b. of neck are never collapse in erect position
c. are thin walled
d. Are innervated
e. of upper limb have no valves

50. Insulin cause:
a. increase utilization of glucose by the cells
b. increase glycogen store by the liver
c. decrease the amino acids and level in blood
d. decrease protein synthesis
e. decrease fatty acids levels in the blood

51. in respiratory acidosis:
a. the blood plasma reaction is acid
b. the kidney excrete an alkaline urine
c. the plasma bicarbonate is decreased
d. the primary defect is hypoventilation
e. cause can be ventilation/perfusion imbalance

52. Toxoplasmosis:
a. the dog is the definite host
b. the parasite exist in 2 forms cyst & oocyst
c. invasion is by the cyst
d. the oocyst is also important in human infection
e. cause cervical lymphadenopathy

53. The following diseases have an incubation period less than 10 days:
a. rabies
b. filariasis
c. dengue
d. yellow fever
e. hepatic amoebiasis

54. Anaerobic bacteria:
a. include gm +ve as well as gm –ve organism
b. produce hyaluronidase
c. bacteroid fragilis is most commonly seen
d. corticosteroid therapy lessen the chance of infection
e. the anaerobic gm –ve bacilli & anaerobic coccci are the major agent in aerobic infection
55. Hepatitis B antigen:
a. consist of Hbs Ag fraction
b. consist of Hbc Ag fraction
c. is only found in the serum sufferers (2-6) weeks after evidence of transaminase activity
d. occurs in 1/10000 of Caucasian
e. may spread in faeces

56. Structure of the simpler virus:
a. in the simpler viruses , the mature virus particle is the virion
b. the protein coat is the capsid
c. the centre is a single molecule of nuclei A
d. the nucleocapsid is the nucleic acid core plus the protein coat
e. capsids are composed of repeating units of one or a small number of polypeptides .

57. the agents of first choice for the organism mentioned are :
a. N. gonorrhea : cephalothin
b. T. palladium : tetracycline
c. C. wechiu : penicillin G
d. E. coli : gentamycin
e. S. aureus : methicillin, oxacillin or norfloxacin

58. consider cell necrosis , a nucleus which shrivels and become more dense is described as:
a. karyorhexis
b. karyolysis
b. pyknosis
d. vacular degeneration
e. condensation

59. Mismatched blood transfusion may lead to:
a. haemoglobinemia
b. haemoglobiuria
c. bilirubinuria
d. polycythaemia
e. renal failure

60. The characteristic reaction to be found in a tubercle usually includes:
a. pus cells
b. siderocyte
c. foreign body giant cells
d. epithelioid cells
e. lymphocytes

61. Multinucleated giant cells are seen in man in association with:
a. cat-scratch disease
b. measles
c. rheumatic fever
d. syphilis
e. TB

65. Necrosis is an important feature of the lesion in:
a. acute anterior poliomyelitis
b. labour pneumonia
c. rheumatic fever
d. tertiary syphilis
e. viral hepatitis

66. the following are true of a normal distribution :
a. 95 % of observations lie within mean +- 1 standard deviation (SD)
b. 2.5 % of observations lie below 2 standard deviation of the mean.
c. student s t-test could be used to compare the date with those of another population which is normally distributed
d. anon-parametric test may may be used to compare the data with those of another population .
e. 99 % of observational lie within 2.6 standard errors of the mean

67. The following statements are correct:
a. the standard error of the mean (SEM) provides an index of the spread of observations around the mean
b. the standard error of the mean is calculated as the square root of variance
c. the standard deviation is generally smaller than the standard error of the mean
d. the standard deviation is an index of the reliability of the mean
e. one advantage of the standard deviation is that it could be manipulated mathematically

68. Hyperpigmenation can occur in pregnancy in the following sites:
a. nipples and areolae
b. umbilicus
c. axillae
d. perineum
e. face
69. Alkalinization of the urine will enhance the excretion of:
a. phencyclidine
b. amiphetamine
c. phenobarbitone
d. aspirin
e. imipramine

70. Which of the following drugs may be administered orally with tetracycline?
a. magnesium trisilicate
b. aluminum hydroxide
c. salicylates
d. indomethacin
e. ferrous sulphate
71. Cell cycle specific antineuplastic agents act during following diseases:
a. DNA synthesis
b. transcription
c. function of the mitotic spindle
d. predna synthetic phase
e. G0 phase

72. Normal amniotic fluid contains:
a. glucose
b. fructose
c. lactose
d. lactic acid
e. citric acid

73. Concerning the physiology of amniotic fluid:
a. at term , it consist only of fetal urine
b. is removed mainly by fetal swallowing
c. is maximum at term
d. 400 – 500 ml are produced /24hrs
e. 5 ml/kg/hr is produced by a term fetus

74. which of the following separate maternal and fetal blood in human placenta :
a. trophoblast
b. maternal vascular endothelium
c. fetal vascular endothelium
d. maternal connective tissue
e. fetal connective tissue 75. Tinea capitis:
a. is very common in adult
b. can be caused by epidermophyton floccosum
c. can be easy treated topically
d. is common in children than adult
e. can occur in an epidemic

76. Hepatitis A is commonly transmitted by:
a. shared needles of drug users
b. faecal oral transfer
c. airborne respiratory droplets
d. blood transfusion
e. insect vectors
[You must be registered and logged in to see this link.]

77. Human papilloma viruses:
a. are typed by DNA hyperdization
b. are associated with condylomata acuminate
c. rarely cause infection in sexually active men
d. are associated with cervical cancer
e. should be subtyped for identification

78. digestive function in a healthy adult involves :
a. decreased production of saliva following parasympathetic activity
b. voluntary oesophageal contraction
c. release of gastrin from the antral portion of the stomach
d. delayed gastric emptying after a fatty meal
e. increased pepsin secretion with vagal stimulation

79. Monocytes in peripheral blood :
a. are less common than eosinophil granulocytes
b. are more common than basophil granulocytes
c. are phagocytic .
d. may migrate into tissues
e. are decreased in pregnancy

80. when blood vessel is injured :
a. factor XII is activated by collagen
b. factor VII is activated by thromboplastin
c. the intrinsic and extrinsic mechanisms share a common pathway following activation of factor VII
d. tissue factors are responsible for the speed of clot formation
e. the fibrin monomer first formed is soluble in plasma
81. comparing the male and female (gynaecoid ) bony pelvis :
a. the subpubic angle is greater in the female
b. in the female the diameter of the acetabulum is less than the distance between the acetabular rim and the symphysis pubis
c. the interspinous distance is greater in the female
d. the greater sciatic notch is larger in the male
e. the curvature of the sacrum is greater in the female

82. The peritoneum:
a. is lined by stratified squamous epithelium
b. has a visceral layer united firmly with underlying organs
c. has a parietal layer innervated by somatic spinal nerves
d. has venous drainage to the portal vein
e. can function as a dialyzing membrane

83. concerning the rectus sheath :
a. above the costal margin rectus abdominis lies on the costal cartilages
b. below the arcuate line the internal oblique splits to enclose rectus abdominis
c. it contains the musculo-phrenic artery
d. it is innervated by the ilio – inguinal nerve
e. pyramidalis is innervated by the subcostal nerve

84. The following drugs are effective systemically when given by mouth:
a. tetracycline hydrochloride
b. gentamicin sulphate
c. streptomycin
d. chloramphenicol
e. nifedipine

85. The administration of chloroquine phosphate may cause:
a. retinopathy
b. bleaching of the hair
c. changes in the cornea
d. deafness
e. photosensitivity

86. Calcification is associated with:
a. chronic renal failure
b. renal calculi
c. parathyroid adenoma
d. hyaline degeneration in uterine fibroids
e. secondary deposits in bone from prostatic carcinoma

87. collagen
a. is the most abundant protein in the body
b. contains hydroxyproline
c. depends in part on non-covalent bonds for its strength
d. is a component of basement membranes
e. is formed from procollagen

88. Wound healings is delayed by:
a. insulin
b. ultraviolet light
c. zinc deficiency
d. low temperature
e. glucocorticoids

89. Prostaglandins:
a. are synthesized from endoperoxides
b. have half live in the circulation of 6 hrs
c. are metabolized in the lungs
d. are synthesized in platelets
e. are synthesized by vascular endothelium

90. compare with cow milk, human milk contains more:
a. lactose
b. casein
c. sodium
d. water
e. ascorbic acid

91. The pelvic surface of the sacrum:
a. gives origin to the piriformis muscle
b. gives origin to the levator ani muscle
c. is broader in the male than in the female
d. transmit the dorsal rami of the sacral nerves
e. is in contact with the anal canal

92. Chromaffin cells:
a. are innervated by pre – ganglionic sympathetic nerve fibers
b. are present in the adrenal cortex
c. are derived from neuro- ectoderm
d. can decarboxylate amino acids
e. are present in celiac ganglia

93. in the fetal circulation:
a. the ductus venosus delivers blood directly into the superior vena cava
b. the umbilical artery returns blood from the placenta
c. the ductus aretriosus carries blood to the lungs
d. blood returning from the lungs is 90 % saturated with oxygen
e. blood from the inferior vena cava is largely directed through the foramen ovale

[You must be registered and logged in to see this link.]

94. The following structures take part in the formation of the anterior fontanelle in the fetal skull:
a. lambdoidal suture
b. occipital suture
c. sagittal suture
d. glabella
e. frontal suture
[You must be registered and logged in to see this link.]

95. The ductus venosus:
a. is part of the embryonic heart
b. is a shunt preventing blood from passing to the fetal lungs
c. gives rise to the legamentum teres
d. caries blood with a higher po2 than umbilical blood
e. is derived from the anterior cardinal vien

96. Successful lactation is:
a. maintained by estrogens
b. maintained by progesterone
c. initiated by a prolactin surge
d. maintained by human placental lactogen
e. inhibited by dopamine

97. in a woman of reproductive age, serum concentration of the following hormones exhibit a recognized pattern of diurnal variation:
a. progesterone
b. melatonin
c. cortisol
d. oestrone
e. follicle stimulating hormone

98. Serum concentration of the following increases during pregnancy:
a. sex hormone binding globulin
b. prolactin
c. total thyroxin
d. follicle stimulating hormone
e. 17 alpha – hydroxyprogesterone

99. Luteinzing hormone:
a. is required for normal corpus luteum survival
b. has a half life in the circulation of 30 hrs
c. is released in pulses
d. in the male stimulates testosterone production
e. plasma concentration are increased in post menopausal women

100. The secretion of growth hormone:
a. occurs in the hypothalamus
b. ceases when the adult stage is reached
c. is decreased during stress
d. is increased during fasting
e. is increased with exercise

101. Parathyroid hormone:
a. decrease the renal secretion of phosphate
b. increase calcium resorption in the bone
c. depresses pituitary activity
d. concentration in blood are raised when the calcium level falls
e. increases renal tubular resorption of calcium

102. Aldosterone:
a. reduce sodium resorption in the proximal convoluted tubules
b. reduces sodium absorption in the descending loop of Henle
c. increase sodium absorption in the distal convoluted tubules
d. increase potassium loss from the tubule
e. increase sodium absorption in the collecting tubule

103. The germination of tetanus spores in a wound is inhibited by :
a. tissue trauma
b. oxygen
c. injection of toxin
d. injection of toxoid
e. removal of devitalized tissue

104. cytomegalovirus :
a. is adenovirus
b. may be cultured readily in cell free media
c. is a cause of cerebral calcification
d. causes haemolytic anaemia in the neonate
e. may be transmitted in the saliva

105. hypokalaemia may be caused by :
a. bendrofluazide
b. digoxin
c. spironolactone
d. carbenoxolone
[You must be registered and logged in to see this link.]

e. amiloride

106. The following are required for haemostatic clot formation:
a. conversion of prothrombin to thrombin
b. platelet phospholipids
c. active conversion of plasminogen to plasmin
d. fibrin degradation product
e. antithrombin

107. Myometrial contractility:
a. is calcium dependant
b. is associated with phosphorylation of myosin light chain
c. is independent of cyclic adenosine monophosphate (cAMP)
d. is mediated by somatic nerves
e. depend on myometrial gap junctions

108. The following values fall within the normal range for the adult female bladder:
a. residual urine of 100 ml
b. voiding volume of 250 ml
c. bladder capacity of 900 ml
d. intravesical pressure rise to less than 10 cm H2O during early filling
e. maximum urine flow rate of 60 ml / second

109. in the testis:
a. maturation from spermatogonia to spermatozoa takes about 29 days
b. sertoli cells can mature into spermatids
c. leydig cells produce inhibin
d. luteinzing hormone inhibits the secretion of testosterone
e. large quantities of fructose are present in the seminal fluid

110. Function of the spleen in the healthy adult includes:
a. erythropoiesis
b. destruction of erythrocytes
c. formation of B lymphocytes
d. phagocytosis of bacteria
e. production erythropoietin


[You must be registered and logged in to see this link.]

111. metronidazole :
a. is effective against giardia lamblia
b. is effective when administered by rectum
c. should not be administered intravenously
d. is usually effective against Entamoeba histolytica
e. interfere with ethanol metabolism


[You must be registered and logged in to see this link.]

112. carcinoma in situ in epithelium ( intraepithelial neoplasia) is characterized by :
a. increased mitotic activity
b. loss of polarity
c. increased adhesiveness to underlying stroma
d. pyknosis
e. increased thickness of epithelium

113. the following provide conclusive evidence of pregnancy in uterine curetting :
a. deciduas compacta
b. arias-stella changes in endometrial glands
c. spiral arterioles
d. plasma cell infiltration
e. chorionic villi

114. Complications of myocardial infarction include:
a. fibrous pericarditis
b. aortic aneurysm
c. ventricular mural thrombi
d. coronary atherosclerosis
e. ventricular aneurysm

115. in the pathogenesis of thrombi :
a. prostacyclin induces platelet aggregation
b. platelets synthesized thromboxane A2
c. thromboxane A2 induces vasoconstriction
d. contact with subendothelial collagen causes platelet


116. the characteristic features of addisonian pernicious anaemia include :
a. leucocytosis
b. inheritance as an autosomal dominant trait
c. a raised corpuscular haemoglobin concentration
d. an increased incidence of gastric neoplasia
e. an increased incidence of primary hypothyroidism

117. immunodeficiency states may be associated with :
a. viral infection of T lymphocytes
b. B cell lymphomas
c. glucocorticoids administration
d. haemolytic disease of the new born
e. untreated Hodgkins lymphoma
[You must be registered and logged in to see this link.]


118. concerning immunoglobulins :
a. IgG contain tow heavy chains
b. IgM is produced before IgG in the immune response
c. IgE is the principle immunoglobulin secreted by mucous membranes
d. IgA is the principle immunoglobulin involved in allergic reactions
e. IgA is secreted in breast milk

119. antibodies :
a. are proteins
b. are formed in the fetus before 12 weeks of intrauterine life
c. have an average molecular weight of around 10,000 Daltons
d. of rhesus type are genetically transmitted
e. are produced by the ribosomes of plasma cells

120. the following disorders have an X linked pattern of inheritance :
a. glucose-6-phosphate dehydrogenase deficiency
b. kleinfelters syndrome
c. adrenogenital syndrome
d. haemophilia B
e. familial hypercholesterolemia

121. the following genetic disorders are inherited as autosomal recessives :
a. Duchene muscular dystrophy
b. Huntingdon’s chorea
c. tay-sachs disease
d. retinoblastoma
e. achondroplasia

122. concerning radiation physics :
a. an electron has a greater mass than a proton
b. a positron has the same charge as an electron
c. a proton has a positive charge
d. a neutron has almost the same mass as a proton
e. the hydrogen nucleus is a neutron

123. the conversion of glucose to lactic acid :
a. occur in a single enzymatic reaction
b. is the only pathway for the synthesis of ATP in the red blood cell
c. is a reversible process in skeletal muscle
d. is inhibited by high cellular concentration of ATP
e. occurs in skeletal muscle when the availability of oxygen is limited

124. normal human seminal fluid :
a. coagulates in vitro
b. conatins sucrose
c. has a pH of 5
d. may contain 15 % of morphologically abnormal spermatozoa
e. is predominantly produced within the testis

125. glucocorticoids :
a. promotes hepatic gluconeogenesis
b. suppress uptake of glucose by the muscle
c. promote protein breakdown
d. promote fat breakdown
e. increase glycolysis in adipose tissue

126. cholesterol :
a.Is essential dietary requirement
b. is present in the plasma membrane of all human cells
c. can not be synthesized by the liver
d. is transported in the circulation bound to albumin
e. is a precursor for the synthesis of steroid hormones .

127. fetal haemoglobin (HbF) :
a. is not formed before 20 weeks of intrauterine life
b. is more resistant than adult haemoglobin to denaturation by alkali
c. in the fetus constitutes 80-90 % of the haemoglobin at term
d. represent less than 5 % of total haemoglobin 8 weeks after birth
e. is increased in adult patients with beta thalassaemia

128. glucagon :
a. is a polypeptide hormone
b. is secreted by beta cells of the pancreatic islets
c. causes muscle glycogenolysis
d. has a half life of 5-10 minutes in the circulation
e. secretion is stimulated by cortisol

129. the female breast :
a. has a base which extend to the ninth rib
b. consist of up to 10 lobes
c. partially overlies the latissmus dorsi muscle
d. receives blood from the internal thoracic artery
e. is supplied in part by supraclavicular nerves

130. labia majora :
a. have a subcutaneous layer of non-striated muscle
b. have a fibro-fatty cord
c. contain the termination of the round ligament of the uterus
d. unite anteriorly forming the fourchette
e. have the openings of the greater vestibular glands on their medial surfaces .

131. the external dural (epidural) space :
a. contains the vertebral venous plexus
b. lie between pia and dura mater
c. contains no cerebrospinal fluid
d. ends at the level of the second lumbar vertebra
e. begins at the level of the first cervical vertebra

132. the following become occluded at or shortly after birth :
a. hepatic veins
b. ductus venosus
c. sinus venosus
d. ductus arteriosus
e. proximal ends of the hypogastric arteries

133. in the cell cycle :
a. DNA synthesis takes place during the M (mitosis) phase
b. RNA synthesis takes place predominantly during the S (synthesis ) phase
c. protein synthesis occurs during the G1 (pre-synthesis ) phase
d. microtubule assembly takes place during the G2 (post-synthesis ) phase
e. chromatids migrate to poles during metaphase

134. the peritoneum :
a. is lined by a stratified squamous epithelium
b. has a visceral layer united firmly with underlying organs
c. has a parietal layer innervated by somatic spinal nerves
d. has venous drainage to the portal vein
e. can function as a dialysing membrane

135. the following are clinical manifestation of infection of Coxsackie’s viruses :
a. aseptic meningitis
b. endometritis
c. epidemic myalgia
d. herpangina
e. pericarditis

136. prolactin secretion :
a. increases during sleep
b. increases during exercise
c. is increased by thyrotropin-realising hormone (TRH)
d. is decreased by dopamine
e. is increased by estrogen

137. in statistics a normal distribution :
a. has 50 % of its values on or above the mean
b. has 95 % of its values within tow standard deviations of the mean
c. has 10 % of its values below the 90th centile
d. has all its values below the third quartile
e. has 99 % of its values within three standard deviations of the mean

138. after the menopause :
a. calcium excretion decreases
b. there is a decreased secretion of follicle stimulating hormone
c. there is an increased secretion of progesterone
d. the vagina pH is increased
e. the endometrium becomes unresponsive to the action of estrogen

139. in the human female :
a. no new ova are formed after birth
b. miotic division is completed by puberty
c. prostaglandins are produced in the endometrium
d. average menstrual blood loss is about 150 ml
e. menstrual flow is maintained from the basal arteries


[You must be registered and logged in to see this link.]

140. the following viruses are commonly transmitted by coitus :
a. hepatitis B virus
b. human papilloma virus
c. herpes simplex virus
d. ebstein bar virus
e. cytomegalovirus

141. therapeutic substances predominantly metabolized by the liver include :
a. gentamicin
b. frusemide
c. morphine
d. phenothiazine
e. chlorpropamide

142. the following drugs are frequently used in the treatment of the named infections :
a. co-trimoxazole __ vaginal candidiasis
b. ampicillin – tuberculosis
c. tetracycline – trachoma
d. chloramphenicol – purulent conjunctivitis
e. trimethoprim – trichomoniasis

143. concerning haemoglobin :
a. haem contain ferric iron
b. the alpha globulin chain contains more than 100 amino acids
c. the affinity of haemoglobin for oxygen is modified by the red blood cell concentration of 2-3 , diphosphoglycerate
d. the oxygen affinity of deoxyhaemoglobin is greater than that of haemoglobin
e. a fall in pH shifts the haemoglobin oxygen dissociation curve to the left

144. Hereditary spherocytosis:
a. is inherited as a sex linked recessive characteristic
b. is associated with a deficiency of red cell pyrineratekinase
c. is associated with decreased fragility of red cells
d. may present in the first month of life
e. is associated with splenomegaly

145. stimulation of alpha adrenergic receptors :
a. does not occur with nor adrenaline
b. in the blood vessels of the skin leads to vasoconstriction
c. increases intestinal peristalsis
d. leads to contraction of erector pili muscles
e. leads to constriction of the pupil

146. epithelioid cells :
a. are transformed tissue macrophages
b. are found in Crohns disease
c. are found in granulation tissue
d. contains phagosomes (secondary lysosomes )
e. may undergo mitosis

147. in chronic infectious disease :
a. there is hyperplasia of the reticulo – endothelial system
b. specific IgG production is diminished
c. humorally mediated hypersensitivity reactions are common
d. anaemia is often of microcytic type
e. serum albumin levels are reduced

148. the following ions are predominantly extracelluar :
a. sodium
b. chloride
c. magnesium
d. phosphate
e. potassium

149. disease transmitted by insect vector :
a. trypansomiasis
b. yellow fever
c. achistosomiasis
d. leishmaniasis
e. filariasis

160. penicillin :
a. may be active against gram negative bacteria
b. interferes with normal cell wall synthesis
c. is inactivated by beta lactamase enzymes
d. is effective against resting bacterial cells
e. is contraindicated in pregnancy

161. the fetal testis :
a. are essential for the formation of the mesonephric (Wolffian) ducts
b. are essential of the formation of the paramesonephric (Mullerian)ducts
c. secretes testosterone
d. arise from the gonadal (genital) ridge
e. differentiate in response to a product of the Y chromosome

162. tetany may occur as a complication of :
a. osteoporosis
b. neonatal hypocalcaemia
c. respiratory alkalosis
d. peripheral neuropathy
e. untreated hyperparathyroidism

163. the tidal volume :
a. is about 700 ml at rest in pregnancy
b. is twice the anatomical dead space
c. together with the residual volume equals the vital capacity
d. is less than alveolar volume
e. is decreased by estrogens

164. the release of catecholamine from the adrenal medulla increases:
a. during sleep
b. when the nerves to the adrenal gland are stimulated
c. when the blood sugar rises
d. during excitement
e. in the presence of a phaeochromocytoma

165. the pineal gland :
a. is stimulated at the anterior end of the third ventricle
b. is innervated by the parasympathetic nervous system
c. produces melatonin
d. may be calcified in the adult
e. is most active during daylight

166. saliva :
a. contain an enzyme which is essential for the complete digestion of stomach
b. contains secretor substances
c. keeps the pH in the mouth between 5.0 and 6.0
d. has an antibacterial action
e. contains mucin

[You must be registered and logged in to see this link.]

167. the secretion of ant6idiuretic hormone is :
a. inhibited by ethyl alcohol
b. controlled by nervous stimuli from the hypothalamus
c. increased by a rising plasma osmolality
d. decreased by a falling plasma volume
e. inhibited by distension of the right atrium

168. mRNA :
a. is a double stranded polymer
b. is transcribed from DNA in the nucleus
c. is not present in reticulocytes
d. contains adenosine
e. is not present in oocytes

169. hypotension causes :
a. diminished urine production
b. inhibition of rennin secretion
c. stimulation of aldosterone secretion
d. stimulation of angiotensin II production
e. increased excretion of sodium

170. in the human the haploid number of chromosomes is found in the :
a. red blood cells
b. blastocyte
c. primary oocytes
d. 1st polar body
e. spermatozoon

171. the following statements relate to plasma proteins :
a. they create an oncotic pressure of 3.3 KPa (25 mm Hg ) across capillary walls
b. they form a major part of the plasma cationic pool
c. all are manufactured in the liver
d. albumin has a lower molecular weight than fibrinogen
e. fibrinogen is freely filtered at the glomerulus

172. in a healthy woman , rennin :
a. is secreted only by the kidney
b. plasma concentration is greater in the pregnant than in the non pregnant state
c. plasma concentration is increased by diuretic therapy
d. converts angiotensinogen into angiotensin II
e. activity is blocked by captopril

173. concerning human parturition :
a. the number of oxytocin receptors in the myometrium increases before the onset of labour
b. in the primigravida , cervical dilation usually precedes cervical effacement
c. the plasma oxytocin concentration increases at the onset of labour
d. oxytocin stimulates the synthesis of prostaglandins within the uterus
e. contraction of the maternal abdominal muscles is essential for spontaneous vaginal delivery

174. recognized effects of pregnancy include :
a. transient impairment of glucose tolerance
b. a raised glomerular filtration rate
c. a raised plasma concentration of free tyrosine
d. a reduced plasma concentration of alkaline phosphatase
e. an increased secretion of prolactin
ahlam qabaja
ahlam qabaja
جني مبتدي
جني مبتدي

عدد المساهمات : 2
نقاط : 2
السٌّمعَة : 5
تاريخ التسجيل : 2014-02-20

Back to top Go down

Back to top

- Similar topics

 
Permissions in this forum:
You cannot reply to topics in this forum